You are on page 1of 30

ESPINA v ZAMORA petitioners or inflicts damages on them, either as

taxpayers4 or as legislators.
FACTS:
B. Constitutional
Pres. Estrada signed into law RA 8762, Retail Trade
Liberalization Act. It repealed RA 1180, which absolutely While Section 19, Article II of the 1987 Constitution
prohibited foreign nationals from engaging in the retail requires the development of a self-reliant and
trade business. RA 8762 allows them to do so. The said independent national economy effectively controlled by
Act allows natural-born Filipino citizens, who had lost Filipino entrepreneurs, it does not impose a policy of
their citizenship and now reside in the Philippines, to Filipino monopoly of the economic environment. The
engage in the retail trade business with the same rights objective is simply to prohibit foreign powers or interests
as Filipinos. from maneuvering our economic policies and ensure that
Filipinos are given preference in all areas of development.
Petitioners, who are all members of the House of
Representatives, filed the petition assailing the While the Constitution mandates a bias in favor of Filipino
constitutionality of the said Act on the following grounds: goods, services, labor and enterprises, it also recognizes
(a) afoul of Sec 9, 19, 20 of Art II of the Constitution; (b) the need for business exchange with the rest of the world
the implementation of R.A. 8762 would lead to alien on the bases of equality and reciprocity and limits
control of the retail trade; (c) foreign retailers would protection of Filipino enterprises only against foreign
crush Filipino retailers and sari-sari store vendors, destroy competition and trade practices that are unfair.
self-employment, and bring about more unemployment;
The 1987 Constitution does not rule out the entry of
(d) World Bank-International Monetary Fund had
foreign investments, goods, and services. While it does
improperly imposed the passage of R.A. 8762 on the
not encourage their unlimited entry into the country, it
government as a condition for the release of certain
does not prohibit them either. In fact, it allows an
loans; (e) clear and present danger that the law would
exchange on the basis of equality and reciprocity,
promote monopolies or combinations in restraint of
frowning only on foreign competition that is unfair. 10 The
trade.
key, as in all economies in the world, is to strike a balance
Respondents countered that petitioners have no legal between protecting local businesses and allowing the
standing; petition does not involve any justifiable entry of foreign investments and services.
controversy; failed to overcome the presumption of
Section 10, Article XII of the 1987 Constitution gives
constitutionality; and the Constitution mandates the
Congress the discretion to reserve to Filipinos certain
regulation but not the prohibition of foreign investments.
areas of investments upon the recommendation of the
ISSUE: NEDA and when the national interest requires. Thus,
Congress can determine what policy to pass and when to
(A) WON petitioners have legal standing to challenge
pass it depending on the economic exigencies. It can
the said Act
enact laws allowing the entry of foreigners into certain
(B) WON RA 8762 is unconstitutional
industries not reserved by the Constitution to Filipino
RULING: citizens. In this case, Congress has decided to open
certain areas of the retail trade business to foreign
A. Does not have legal standing. investments instead of reserving them exclusively to
The long settled rule is that he who challenges the validity Filipino citizens. The NEDA has not opposed such policy.
of a law must have a standing to do so. 1 Legal standing Here, to the extent that R.A. 8762, the Retail Trade
or locus standi refers to the right of a party to come to a Liberalization Act, lessens the restraint on the foreigners’
court of justice and make such a challenge. More right to property or to engage in an ordinarily lawful
particularly, standing refers to his personal and business, it cannot be said that the law amounts to a
substantial interest in that he has suffered or will suffer denial of the Filipinos’ right to property and to due
direct injury as a result of the passage of that law. process of law. Filipinos continue to have the right to
Here, there is no clear showing that the implementation engage in the kinds of retail business to which the law in
of the Retail Trade Liberalization Act prejudices question has permitted the entry of foreign investors.
The law itself has provided strict safeguards on foreign Section 12. The State shall promote the preferential use
participation in that business. Thus – of Filipino labor, domestic materials and locally produced
goods, and adopt measures that help make them
First, aliens can only engage in retail trade business
competitive.
subject to the categories above-enumerated; Second,
only nationals from, or juridical entities formed or Section 13. The State shall pursue a trade policy that
incorporated in countries which allow the entry of Filipino serves the general welfare and utilizes all forms and
retailers shall be allowed to engage in retail trade arrangements of exchange on the basis of equality and
business; and Third, qualified foreign retailers shall not be reciprocity
allowed to engage in certain retailing activities outside
their accredited stores through the use of mobile or
rolling stores or carts, the use of sales representatives, G.R. No. 158540             July 8, 2004
door-to-door selling, restaurants and sari-sari stores and
SOUTHERN CROSS CEMENT CORPORATION
such other similar retailing activities. vs.
THE PHILIPPINE CEMENT MANUFACTURERS CORP.,
Provision:
THE SECRETARY OF THE DEPARTMENT OF TRADE
ART II & INDUSTRY, THE SECRETARY OF THE
DEPARTMENT OF FINANCE, and THE
Section 9. The State shall promote a just and dynamic COMMISSIONER OF THE BUREAU OF CUSTOMS
social order that will ensure the prosperity and TINGA, J.:
independence of the nation and free the people from
poverty through policies that provide adequate social
services, promote full employment, a rising standard of FACTS:
living, and an improved quality of life for all.
The Philippines, enacted laws soon after it joined the
Section 19. The State shall develop a self-reliant and General Agreement on Tariff and Trade (GATT) and the World
independent national economy effectively controlled by Trade Organization (WTO) Agreement. One of these laws is
Rep. Act No. 8800, also known as the Safeguard Measures Act
Filipinos.
("SMA"). The SMA provides the structure and mechanics for
Section 20. The State recognizes the indispensable role of the imposition of emergency measures, including tariffs, to
protect domestic industries and producers from increased
the private sector, encourages private enterprise, and
imports which inflict or could inflict serious injury on them.
provides incentives to needed investments.
Petitioner Southern Cross Cement Corporation
ART XII ("Southern Cross") is a domestic corporation engaged in the
business of cement manufacturing, production, importation and
Section 10. The Congress shall, upon recommendation of exportation. Its principal stockholders are Taiheiyo Cement
the economic and planning agency, when the national Corporation and Tokuyama Corporation, purportedly the largest
interest dictates, reserve to citizens of the Philippines or cement manufacturers in Japan. Private respondent, on the other
to corporations or associations at least sixty per centum hand, Philippine Cement Manufacturers Corporation
of whose capital is owned by such citizens, or such higher ("Philcemcor") is an association of domestic cement
manufacturers. It has eighteen (18) members. However, it
percentage as Congress may prescribe, certain areas of
appears that considerable equity holdings, if not controlling
investments. The Congress shall enact measures that will interests in at least twelve (12) of its member-corporations, were
encourage the formation and operation of enterprises acquired by the three largest cement manufacturers in the
whose capital is wholly owned by Filipinos. world, 

In the grant of rights, privileges, and concessions covering On 22 May 2001, respondent Department of Trade and
the national economy and patrimony, the State shall give Industry accepted an application from Philcemcor, alleging that
the importation of gray Portland cement in increased quantities
preference to qualified Filipinos.
has caused declines in domestic production, capacity utilization,
The State shall regulate and exercise authority over market share, sales and employment; as well as caused
foreign investments within its national jurisdiction and in depressed local prices. Accordingly, Philcemcor sought the
imposition a definitive safeguard measures on the import of
accordance with its national goals and priorities.
cement pursuant to the Safeguard Measures Act.
After preliminary investigation, the Bureau of Import legislative intent is to grant the DTI Secretary the power to
Services of the DTI, determined that critical circumstances make a final decision on the Tariff Commission's
existed justifying the imposition of provisional measures. On recommendation.
7 November 2001, the DTI issued an Order, imposing a
provisional measure equivalent to Twenty Pesos and Sixty Southern Cross filed the present petition, assailing the
Centavos (P20.60) per forty (40) kilogram bag on all appellate court's Decision for departing from the accepted and
importations of gray Portland cement for a period not exceeding usual course of judicial proceedings, and not deciding the
two hundred (200) days from the date of issuance by the Bureau substantial questions in accordance with law and jurisprudence.
of Customs (BOC) of the implementing Customs Memorandum The timely filing of Southern Cross's petition before this Court
Order. necessarily prevented the Court of Appeals Decision from
becoming final.
The Tariff Commission, on 19 November 2001,
received a request from the DTI for a formal investigation to On 25 June 2003, the DTI Secretary issued a
determine whether or not to impose a definitive safeguard new Decision, He made a determination that, contrary to the
measure on imports of gray Portland cement, pursuant to findings of the Tariff Commission, the local cement industry
Section 9 of the SMA and its Implementing Rules and had suffered serious injury as a result of the import surges.
Regulations. The Tariff Commission after arriving at their Accordingly, he imposed a definitive safeguard measure on the
conclusions, made a recommendation that the elements of importation of gray Portland cement, in the form of a definitive
serious injury and imminent threat of serious injury not safeguard duty in the amount of P20.60/40 kg. bag for three
having been established, it is hereby recommended that no years on imported gray Portland Cement.
definitive general safeguard measure be imposed on the
importation of gray Portland cement.
ISSUE:
After reviewing the report, then DTI Secretary Manuel
Roxas II disagreed with the conclusion of the Tariff Whether or not the DTI Secretary is bound to adopt the
Commission. In view of this disagreement, the DTI requested an negative recommendation of the Tariff Commission on the
opinion from the Department of Justice (DOJ) on the DTI application for safeguard measure.
Secretary’s scope of options in acting on the Commission’s
recommendations. DOJ Secretary Hernando Perez rendered an
opinion stating that Section 13 of the SMA precluded a review RULING:
by the DTI Secretary of the Tariff Commissions negative
finding, or finding that a definitive safeguard measure should YES.
not be imposed. Hence, DTI then denied application for
Undoubtedly, Section 13 prescribes certain limitations and
safeguard measures against the importation of gray Portland
restrictions before general safeguard measures may be imposed.
cement.
However, the most fundamental restriction on the DTI
Philcemcor filed with the Court of Appeals a Petition Secretary's power in that respect is contained in Section 5 of
for Certiorari, Prohibition and Mandamus seeking to set aside the SMA that there should first be a positive final
the DTI Decision, as well as the Tariff Commissions Report. On determination of the Tariff Commission.
the other hand, Southern Cross filed its Comment arguing that
Sec. 5. Conditions for the Application of General
the Court of Appeals had no jurisdiction over Philcemcors
Safeguard Measures. – The Secretary shall apply a
Petition, for it is on the Court of Tax Appeals (CTA) that the
general safeguard measure upon a positive final
SMA conferred jurisdiction to review rulings of the Secretary in
determination of the [Tariff] Commission that a
connection with the imposition of a safeguard measure.
product is being imported into the country in increased
COURT OF APPEALS RULING quantities, whether absolute or relative to the domestic
production, as to be a substantial cause of serious
The appellate court ruled that it had jurisdiction over injury or threat thereof to the domestic industry;
the petition for certiorari since it alleged grave abuse of however, in the case of non-agricultural products, the
discretion. It refused to annul the findings of the Tariff Secretary shall first establish that the application of
Commission, citing the rule that factual findings of such safeguard measures will be in the public interest.
administrative agencies are binding upon the courts and its (emphasis supplied)
corollary, that courts should not interfere in matters addressed to
the sound discretion and coming under the special technical The plain meaning of Section 5 shows that it is the
knowledge and training of such agencies. Nevertheless, it held Tariff Commission that has the power to make a "positive final
that the DTI Secretary is not bound by the factual findings determination." This power lodged in the Tariff Commission,
of the Tariff Commission since such findings are merely must be distinguished from the power to impose the general
recommendatory and they fall within the ambit of the safeguard measure which is properly vested on the DTI
Secretary's discretionary review. It determined that the Secretary.
All in all, there are two condition precedents that must to the DTI Secretary, with the instruction that the DTI Secretary
be satisfied before the DTI Secretary may impose a general may impose a general safeguard measure even if there is no
safeguard measure on grey Portland cement. First, there must positive final determination from the Tariff Commission. More
be a positive final determination by the Tariff Commission that crucially, the Court of Appeals could not have acquired
a product is being imported into the country in increased jurisdiction over Philcemcor's petition for certiorari in the first
quantities, as to be a substantial cause of serious injury or place, as Section 29 of the SMA properly vests jurisdiction on
threat to the domestic industry. Second, in the case of non- the CTA. Consequently, the assailed Decision is an absolute
agricultural products the Secretary must establish that the nullity, and we declare it as such. It is clear then that the 25 June
application of such safeguard measures is in the public interest. 2003 Decision of the DTI Secretary is a product of the
void Decision, it being an attempt to carry out such null
Section 5 plainly evinces legislative intent to restrict the DTI judgment.
Secretary's power to impose a general safeguard measure by
preconditioning such imposition on a positive determination
by the Tariff Commission. Section 28(2), Article VI of the
1987 Constitution confirms the delegation of legislative power,
yet ensures that the prerogative of Congress to impose
limitations and restrictions on the executive exercise of this G.R. No. 86953
power: MARINE RADIO COMMUNICATIONS ASSOCIATION
OF THE PHILIPPINES, INC. (MARCAPI), et al.,
The Congress may, by law, authorize the President to
petitioners,
fix within specified limits, and subject to such
limitations and restrictions as it may impose, tariff vs.
rates, import and export quotas, tonnage and wharfage
RAINERIO O. REYES, in his capacity as Secretary of the
dues, and other duties or imposts within the framework
Department of Transportation and Communications (DOTC), et
of the national development program of the
al., respondents.
Government.
SARMIENTO, J.:
This delegation of the taxation power by the legislative
to the executive is authorized by the Constitution itself. At the FACTS:
same time, the Constitution also grants the delegating authority The petitioners are Filipino entrepreneurs deeply
(Congress) the right to impose restrictions and limitations on the involved in the business of marine radio communications in the
taxation power delegated to the President.  The restrictions and country. They are also operators of "shore-to-ship and ship-to-
limitations imposed by Congress take on the mantle of a shore public marine coastal radio stations”, and are holders of
constitutional command, which the executive branch is obliged certificates of public convenience issued by the NTC. Among
to observe. other things, they handle correspondence between vessel
The SMA empowered the DTI Secretary, as alter passengers or crew and the public.
ego of the President, to impose definitive general safeguard Sometime in 1988, in light of the tragic incident of the
measures, which basically are tariff imposts of the type spoken sinking of MV Dona Paz, the DOTC designed an P880-million
of in the Constitution. However, the law did not grant him full, maritime coastal communications system project, designed to
uninhibited discretion to impose such measures. The DTI provide, among other things, ship-to-shore and shore-to-ship
Secretary authority is derived from the SMA; it does not flow public corresponding, free of charge.
from any inherent executive power. Thus, the limitations
After a failed “appeal” letter addressed to then-DOTC
imposed by Section 5 are absolute, warranted as they are by a
constitutional fiat. Secretary Reyes, the petitioners brought the instant suit, alleging
that Secretary Reyes had been guilty of grave abuse of
Moreover, the DTI Secretary does not have the power discretion. The petitioners hold that the Department cannot
to review the findings of the Tariff Commission for it is not compete in the business of public correspondence, and rely on
subordinate to the Department of Trade and Industry, but of the provisions of Section 20, of Article II, of the Constitution,
the National Economic Development Authority, an which states:
independent planning agency of the government of co-equal
rank as the DTI. DTI Secretary generally cannot exercise Sec. 20. The State recognizes the
review authority over actions of the Tariff Commission. Neither indispensable role of the private sector,
does the SMA specifically authorize the DTI Secretary to alter, encourages private enterprise, and
amend or modify in any way the determination made by the provides incentives to needed
Tariff Commission. The most that the DTI Secretary could do to investments.
express displeasure over the Tariff Commission's actions is to
ignore its recommendation, but not its determination. ISSUE:
WoN Section 20, Article II of the Constitutional
The Court of Appeals erred in remanding the case back
operates to prohibit the government from entering the industry 1978, with a reservation annotated on the back of its application
of marine radio communications. that it is not waiving its rights over its mining claims until the
validity of Presidential Decree No. 1214 shall have been passed
RULING: upon by this Court. Three (3) days before filing the disputed
NO. The duty of the State is predominantly "to serve mining lease application, petitioner filed this special civil action
the people”, and to "promote a just and dynamic social order” for certiorari and prohibition. Petitioner assails
through policies that provide adequate social services and an
improved quality of life for all. Petitioner avers that its fifty (50) mining claims had
already been declared as its own private and exclusive property
The petitioners cannot legitimately rely on the in final judgments rendered by the CFI of Camarines Norte in
provisions of Section 20, of Article II, of the Constitution to land registration proceedings initiated by third persons, such as:
defeat the act complained of. The mandate recognizing the
indispensable role of the private sector is no more than an 1. September 1951 land title application by a certain
acknowledgment of the importance of private initiative in Gervacio Liwanag, where the Director of Mines
building the nation. However, it is not a call for official opposed the grant of said application because
petitioners had already located and perfected its
abdication of duty to citizenry.
mining claims over the area applied for.
The novel provisions of the Charter prescribing private
sector participation, especially in the field of economic activity, 2. LRC Case No. 240, filed 11 July 1960, by one
Antonio Astudillo and decided in 1974 against said
come, indeed, no more as responses to State monopoly of
applicant, in which, petitioner's mining claims
economic forces which has unfairly kept individual initiative were described as vested property outside the
from the economic processes and has held back competitiveness jurisdiction of the Director of Mines. 
in the market. The Constitution does not bar, however, the
Government from undertaking its own initiatives, especially Respondent: Petitioner has no standing to file the instant petition
in the domain of public service, and neither does it repudiate its as it failed to fully exhaust administrative remedies. There is a
primacy as chief economic caretaker of the nation. pending appeal of the petitioner at the Office of the President, of
the ruling of the respondent Secretary of Natural Resources in a
The petition is dismissed. DNR Case, which upheld the decision that forty four (44) out of
petitioner's fifty (50) mining claims were void for lack of valid
ADDITIONAL NOTES: The Court is not of the thinking that
"tie points" as required under the Philippine Bill of 1902, and
the act complained of is equivalent to a taking without just that all the mining claims had already been abandoned and
compensation. While it is true that when the owner is deprived cancelled, for petitioner's non-compliance with the legal
of the ordinary and beneficial use of his property or of its value requirements of the same Phil. Bill of 1902 and Executive Order
by its being diverted to public use, there is taking within the No. 141. Respondents further contend that, even
constitutional sense, it does not seem to mean that the DOTC, assuming arguendo that petitioner's mining claims were valid at
by providing for free public correspondence, is guilty of an the outset, if they are deemed abandoned and cancelled due to
non-compliance with the legal requirements for maintaining a
uncompensated taking. Rather, the Government merely built a
perfected mining claim, under the provisions of the Philippine
bridge that made the boat obsolete, although not entirely useless. Bill of 1902.
Certainly, the owner of the boat cannot charge the builder of the
bridge for lost income. And certainly, the Government has all Petitioner, on the other hand, would rebut respondents' argument
the right to build the bridge. by declaring that it already had a vested right over its mining
claims even before Presidential Decree No. 1214, following the
rulings in McDaniel v. Apacible 7 and Gold Creek Mining Corp.
v. Rodriguez. 

ISSUE: Whether Presidential Decree No. 1214 is


G.R. No. L-49109 December 1, 1987 unconstitutional in that it amounts to a deprivation of property
without due process of law.
SANTA ROSA MINING COMPANY, INC., petitioner, vs.
HON. MINISTER OF NATURAL RESOURCES JOSE J. RULING:
LEIDO, JR. AND DIRECTOR OF MINES JUANITO C.
FERNANDEZ, respondents. Certiorari: It is premature for the Court to now make a finding
on the matter of whether petitioner had abandoned its mining
FACTS: On 14 October 1977, Presidential Decree No. 1214 claims. Until petitioner's appeal shall have been decided by the
was issued, requiring holders of subsisting and valid patentable Office of the President, where it is pending, petitioner's attempt
mining claims located under the provisions of the Philippine Bill to seek judicial recognition of the continuing validity of its
of 1902 to file a mining lease application within one (1) year mining claims, cannot be entertained by the Court.
from the approval of the Decree. Petitioner accordingly filed
a mining lease application, but "under protest," on 13 October
Mining Claims:  The decisions of the Court of First Instance of natural resources shall be granted for a period
Camarines Norte, relied upon by petitioner, do not foreclose a exceeding twenty-five years, renewable for
proceeding, such as DNR Case No. 4140, to determine whether not more than twenty-five years, except as to
petitioner's unpatented mining claims have remained valid and water rights for irrigation, water supply,
subsisting. fisheries, or industrial uses other than the
development of water power, in which cases,
The cases cited by petitioner, true enough, recognize the right of beneficial use may be the measure and the
a locator of a mining claim as a property right. This right, limit of the grant.
however, is not absolute. It is merely a possessory right, more
so, in this case, where petitioner's claims are still unpatented. The same constitutional mandate is found in Sec. 2, Art. XII of
They can be lost through abandonment or forfeiture or they may the 1987 Constitution, which declares:
be revoked for valid legal grounds.
All lands of the public domain, waters,
McDaniel v. Apacible: It confirms that a valid mining claim may minerals, coal, petroleum, and other mineral
still be lost through abandonment or forfeiture. oils, all forces of potential energy, fisheries,
forests or timber, wildlife, flora and fauna. and
Gold Creek Mining Corp. v. Rodriguez: In that case, what was other natural resources are owned by the State.
in issue was Gold Creek's right to a patent over its mining With the exception of agricultural lands, all
claim, after compliance with all legal requirements for a patent. other natural resources shall not be alienated.
In the present case, no application for patent is in issue, although The exploration, development, and utilization
as a holder of patentable mining claims petitioner could have of natural resources shall be under the full
applied for one during all these years but inexplicably did not do control and supervision of the State.
so. In Gold Creek, no finding of abandonment was ever made
against the mining claimant as to deprive it of the initial WHEREFORE, premises considered, the petition is hereby
privilege given by virtue of its location; on the other hand, such DISMISSED.
a finding has been made in petitioner's case.

Constitutionality: Presidential Decree No. 1214 is not


unconstitutional. It is a valid exercise of the sovereign power of SAN MIGUEL CORPORATION v. COURT OF APPEALS
the State, as owner, over lands of the public domain, of which and DIRECTOR OF LANDS
petitioner's mining claims still form a part, and over the
G.R. No. 57667. May 28, 1990
patrimony of the nation, of which mineral deposits are a
valuable asset. It may be underscored, in this connection, that FACTS: This is a petition for review on certiorari where
the Decree does not cover all mining claims located under the petitioner San Miguel Corporation who purchased Lot 684
Phil. Bill of 1902, but only those claims over which their from Silverio Perez, seeks the reversal of the decision of
locators had failed to obtain a patent. And even then, such the Court of Appeals denying its application for registration
locators may still avail of the renewable twenty-five year (25) of the said land in view of its failure to show entitlement
lease prescribed by Pres. Decree No. 463, the Mineral thereto.
Development Resources Decree of 1974.
The Solicitor General opposed and appealed the
Mere location does not mean absolute ownership over the application contending that the land in question is part of
affected land or the mining claim. It merely segregates the public domain and that petitioner being a private
located land or area from the public domain by barring other corporation, under Section 11, Article XIV of the
would-be locators from locating the same and appropriating for Constitution, is disualified from holding alienable lands of
themselves the minerals found therein. To rule otherwise would the public domain.
imply that location is all that is needed The lower court rendered a decision granting the
to acquire and maintain rights over a located mining claim. application for registration and adjudicating the property in
Presidential Decree No. 1214 is in accord with Sec. 8, Art. XIV favor of SMC. The Solicitor General appealed to the Court
of the 1973 Constitution which states: of Appeals. In its decision of March 23, 1981, said court
reversed the decision of the lower court and declared the
All lands of the public domain, waters, parcel of land involved as public land. Hence, the instant
minerals, coal, petroleum, and other mineral petition with SMC submitting the following alleged "grave
oils, all forces of potential energy, fisheries, errors" of the Court of Appeals for this Court’s resolution:
wildlife, and other natural resources of the (1) the Court of Appeals’ failure to hold that "prescription is
Philippines belong to the State. With the a mode of acquiring title or ownership of land and that the
exception of agricultural, industrial or title thus acquired is registrable" ; (2) the Court of Appeals’
commercial, residential and resettlement lands disregard of SMC’s evidence "not on the basis of
of the public domain, natural resources shall controverting evidence but on the basis of unfounded
not be alienated, and no license, concession, suppositions and conjectures," and (3) the Court of
or lease for the exploration, development, Appeals’ reversal of the factual findings of the trial court
exploitation, or utilization of any of the which had the opportunity of observing the demeanor and
sincerity of the witnesses.
Petitioner claims that its predecessor-in-interest had open,
exclusive and undisputed possession of the land in G.R. No. 149927            
question based on documentary evidence of tax
declarations and receipts, and testimonial evidence of March 30, 2004
vendor Silverio Perez.
426 SCRA 517
ISSUE:
(1) Whether or not tax declarations and receipts are
conclusive evidence of ownership or right of Republic vs. Rosemoor Mining and Development
possession over a piece of land. Corporation

(2) Whether or not the factual findings of the trial PRINCIPLE:


courts may be reversed by the Court of Appeals.
A mining license that contravenes a mandatory provision
of the law under which it is granted is void. Being a mere
privilege, a license does not vest absolute rights in the
(3) What is the period prescribed by law to prove the
holder. Thus, without offending the due process and the
conclusive ownership over an alienable public
land. non-impairment clauses of the Constitution, it can be
revoked by the State in the public interest.
HELD:
FACTS:
(1) No, documentary evidence of tax declarations and
receipts are not conclusive evidence of ownership Petitioners were granted permission to prospect for
or right of possession over a piece of land but marble deposits in the mountain of Biak-na-Bato in San
mere indicia of a claim of ownership. They only Miguel, Bulacan. After succeeding in discovering marble
become strong evidence of ownership of land deposits of high quality and commercial quantities, they
acquired by prescription when accompanied by applied with the Mines and Geosciences Bureau for the
proof of actual possession. Also, the testimony of issuance of license to exploit said marble deposits.
vendor Silverio Perez as proof of actual (Syempre, pera ‘yun besh.)
possession is weak and was not corroborated by
other witnesses. Since PD 463 limits that a quarry license should cover not
more than 100 hectares in any given province,
respondents applied 4 separate applications, 81 hectares
(2) Yes. Suffice it to state that while trial courts may each, applying therefore for a total of 330.3062 hectares.
have the right to observe the demeanor of the (Lusot!)
witnesses, their factual findings may nonetheless
be reversed by the Court of Appeals, the appellate After compliance with the requirements, License No. 33
court vested by law to resolve both the legal and was issued. (Yay! Dami kikitain nitong respondent na ‘to.)
factual issues, if, by the evidence on record, it
appears that the trial court involved erred. (Kaya lang may umepal) Respondent Ernesto R. Maceda
was appointed Minister of the Department of Energy and
Natural Resources (DENR), petitioners’ License No. 33
was cancelled by him through a letter to the petitioners
(3) Open, exclusive and undisputed possession of
alienable public land for the period prescribed by dated September 6, 1986. Because of said cancellation, a
law creates the legal fiction whereby the land, petition was filed to assail the same. (Palag ‘to si
upon completion of the requisite period ipso jure respondent, bali Miners vs. DENR na!)
and without the need of judicial or other sanction,
RULING OF RTC:
ceases to be public land and becomes private
property. Such open, continuous, exclusive and 1. Privilege granted under respondents’ license had
notorious occupation of the disputed properties for already ripened into a property right, which was protected
more than 30 years must, however, be
under the due process clause of the Constitution. Such
conclusively established. This quantum of proof is
right was supposedly violated when the license was
necessary to avoid the erroneous validation of
actually fictitious claims of possession over the cancelled without notice and hearing.
property in dispute. 2. The cancellation was said to be unjustified, because the
area that could be covered by the four separate
applications of respondents was 400 hectares.
3. Finally, according to the RTC, Proclamation No. 84, RA 7942 or the Philippine Mining Act of 1995 embodies
which confirmed the cancellation of the license, was an ex the new constitutional mandate. It has repealed or
post facto law; as such, it violated Section 3 of Article amended all laws that are inconsistent with any of its
XVIII of the 1987 Constitution. provisions.

(Syempre appeal ‘to si DENR, di magpapatalo) Petitioner


appealed lower court’s ruling.
HOWEVER:
RULING OF COURT OF APPEALS:
Section 2 of Article XII of the 1987 Constitution does not
(Sustained ni CA ang ruling ni RTC) apply retroactively to a "license, concession or lease"
granted by the government under the 1973 Constitution or
1. The grant of the quarry license covering 330.3062 before the effectivity of the 1987 Constitution.
hectares to respondents was authorized by law, because
the license was embraced by four (4) separate While RA 7942 has expressly repealed provisions of
applications -- each for an area of 81 hectares. mining laws that are inconsistent with its own, it
nonetheless respects previously issued valid and existing
2. The cancellation of respondents’ license without notice licenses, as follows:
and hearing was tantamount to a deprivation of property
without due process of law. 1. Mineral reservations are subject to valid existing
mining/quarrying rights. (Section 5, RA 7942)
3. Under the clause in the Constitution dealing with the
non-impairment of obligations and contracts, respondents’ 2. Periodic Review of Existing Mineral Reservations to be
license must be respected by the State done without prejudice to prior existing rights. (Section 7,
RA 7942)
(Edi ngayon nagpetition na ‘to si petitioner (DENR), talo
siya RTC and CA eh) 3. Areas Open to Mining Operations are subject to any
existing rights or reservations and prior agreements of all
ISSUES: parties (Section 18, RA 7942)
1. Is the license valid? Petitioners submit that the license clearly contravenes
2. Is Proclamation No. 84 issued by then President Section 69 of PD 463, because it exceeds the maximum
Corazon Aquino is valid. (This is the law reverting lands area that may be granted. This incipient violation,
back to formal status as part of Biak-na-Bato National according to them, renders the license void ab initio.
Park.) Respondents, on the other hand, argue that the license
was validly granted, because it was covered by four
separate applications for areas of 81 hectares each.
SC RULING: The Petition has merit.

SC RULING: VALIDITY OF LICENSE:


The terms of the license allowed the corporation to extract
Respondents contend that the Petition has no legal basis, and dispose of marbleized limestone from a 330.3062-
because PD 463 has already been repealed by RA 7942 hectare land in San Miguel, Bulacan. The license is,
increasing coverage for explorations. In effect, they ask for however, subject to the terms and conditions of PD 463,
the dismissal of the Petition on the ground of mootness. the governing law at the time it was granted; as well as to
the rules and regulations promulgated thereunder.
PD 463, pertained to the old system of exploration of
natural resources through licenses. While these By the same token, Proclamation No. 2204 -- which
arrangements were provided under the 1935 and the awarded to Rosemoor the right of development,
1973 Constitutions, they have been omitted by Section 2 exploitation, and utilization of the mineral site -- expressly
of Article XII of the 1987 Constitution. cautioned that the grant was subject to "existing policies,
laws, rules and regulations.
With the shift of constitutional policy toward "full control
and supervision of the State" over natural resources, The license was thus subject to Section 69 of PD 463,
jurisprudence declared the provisions of PD 463 as which limits that a quarry license shall cover an area of not
contrary to or violative of the express mandate of the 1987 more than one hundred (100) hectares in any one
Constitution. The said provisions dealt with the lease of province and not more than one thousand (1,000)
mining claims; quarry permits or licenses covering hectares in the entire Philippines.
privately owned or public lands; and other related
provisions on lease, licenses and permits. The language of PD 463 is clear. It states in categorical
and mandatory terms that a quarry license, like that of necessarily vested in the State as owner of all mineral
respondents, should cover a maximum of 100 hectares in resources. That determination was based on policy
any given province. This law neither provides any considerations formally enunciated in the letter dated
exception nor makes any reference to the number of September 15, 1986, issued by then Minister Maceda and,
applications for a license. Section 69 of PD 463 must be subsequently, by the President through Proclamation No.
taken to mean exactly what it says. 84.

Clearly, the intent of the law would be brazenly Moreover, granting that respondents’ license is valid, it
circumvented by ruling that a license may cover an area can still be validly revoked by the State in the exercise of
exceeding the maximum by the mere expediency of filing police power.
several applications. Such ruling would indirectly permit an
act that is directly prohibited by the law.
Proclamation No. 84 cannot be stigmatized as a violation
of the non-impairment clause. As pointed out earlier,
SC RULING: VALIDITY OF PROCLAMATION NO. 84: respondents’ license is not a contract to which the
protection accorded by the non-impairment clause may
Petitioners also argue that the license was validly declared extend.
a nullity and consequently withdrawn or terminated. In a
letter dated September 15, 1986, respondents were The court did not sustain the argument that Proclamation
informed by then Minister Ernesto M. Maceda that their No. 84 is a bill of attainder; that is, a "legislative act which
license had illegally been issued, because it violated inflicts punishment without judicial trial." Its declaration that
Section 69 of PD 463; and that there was no more public QLP No. 33 is a patent nullity is certainly not a declaration
interest served by the continued existence or renewal of of guilt. Neither is the cancellation of the license a
the license. punishment within the purview of the constitutional
proscription against bills of attainder.
The latter reason, they added, was confirmed by the
language of Proclamation No. 84. According to this law, Too, there is no merit in the argument that the
public interest would be served by reverting the parcel of proclamation is an ex post facto law. There are six
land that was excluded by Proclamation No. 2204 to the recognized instances when a law is considered as such
former status of that land as part of the Biak-na-Bato and Proclamation No. 84 does not fall under any of the
national park. enumerated categories; hence, it is not an ex post facto
law.
They also contend that Section 74 of PD 463 would not
apply, because Minister Maceda’s letter did not cancel or It is settled that an ex post facto law is limited in its scope
revoke QLP No. 33, but merely declared the latter’s nullity. only to matters criminal in nature. Proclamation 84, which
They further argue that respondents waived notice and merely restored the area excluded from the Biak-na-Bato
hearing in their application for the license. national park by canceling respondents’ license, is clearly
not penal in character.
On the other hand, respondents submit that, as provided
for in Section 74 of PD 463, their right to due process was
violated when their license was cancelled without notice
and hearing. They likewise contend that Proclamation No. PETITION WAS GRANTED. RULIGN OF CA WAS SET
84 is not valid for the following reasons: 1) it violates the ASIDE.
clause on the non-impairment of contracts; 2) it is an ex
post facto law and/or a bill of attainder; and 3) it was
issued by the President after the effectivity of the 1987
Constitution. La Bugal-B’Laan Tribal Association, Inc. vs. Ramos
445 SCRA 1, G.R. No. 127882 December 1, 2004
In line with the foregoing jurisprudence, respondents’ FACTS:
license may be revoked or rescinded by executive action
when the national interest so requires, because it is not a  The constitutional provision allowing the President to
contract, property or a property right protected by the due enter into FTAA is a exception to the rule that
process clause of the Constitution.29 Respondents participation in the nation’s natural resources is reserved
themselves acknowledge this condition of the grant under exclusively to Filipinos. Provision must be construed
paragraph 7 of QLP No. 33, which states that strictly against their enjoyment by non-Filipinos.
permits/licenses may be revoked or cancelled at any time
by the Director of Mines and Geo-Sciences when, in his  RA 7942 (The Philippine Mining Act) took effect on April
opinion public interests so require. 9, 1995. Before the effectivity of RA 7942, or on March 30,
1995, the President signed a Financial and Technical
The determination of what is in the public interest is
Assistance Agreement (FTAA) with WMCP, a corporation Regalian Doctrine which states that ―All lands of the
organized under Philippine laws, covering close to public domain, waters, minerals, coal, petroleum, and
100,000 hectares of land in South Cotabato, Sultan other minerals, coal, petroleum, and other mineral oils, all
Kudarat, Davao del Sur and North Cotabato. On August forces of potential energy, fisheries, forests or timber,
15, 1995, the Environment Secretary Victor Ramos issued wildlife, flora and fauna, and other natural resources are
DENR Administrative Order 95-23, which was later owned by the State. The same section also states that, ―the
repealed by DENR Administrative Order 96-40, adopted exploration and development and utilization of natural
on December 20, 1996. resources shall be under the full control and supervision
of the State.
 Petitioners prayed that RA 7942, its implementing rules, Conspicuously absent in Section 2 is the provision in the
and the FTAA between the government and WMCP be 1935 and 1973 Constitution authorizing the State to grant
declared unconstitutional on ground that they allow fully licenses, concessions, or leases for the exploration,
foreign owned corporations like WMCP to exploit, exploitation, development, or utilization of natural
explore and develop Philippine mineral resources in resources. By such omission, the utilization of inalienable
contravention of Article XII Section 2 paragraphs 2 and 4 lands of the public domain through license, concession or
of the Charter. lease is no longer allowed under the 1987 Constitution.
 In January 2001, WMC – a publicly listed Australian Under the concession system, the concessionaire makes a
mining and exploration company – sold its whole stake in direct equity investment for the purpose of exploiting a
WMCP to Sagittarius Mines, 60% of which is owned by particular natural resource within a given area. The
Filipinos while 40% of which is owned by Indophil concession amounts to complete control by the
Resources, an Australian company. DENR approved the concessionaire over the country‘s natural resource, for it is
transfer and registration of the FTAA in Sagittarius‘ name given exclusive and plenary rights to exploit a particular
but Lepanto Consolidated assailed the same. The latter resource at the point of extraction.
case is still pending before the Court of Appeals. The 1987 Constitution, moreover, has deleted the phrase
―management or other forms of assistance in the 1973
 EO 279, issued by former President Aquino on July 25, Charter. The present Constitution now allows only
1987, authorizes the DENR to accept, consider and ―technical and financial assistance. The management and
evaluate proposals from foreign owned corporations or the operation of the mining activities by foreign
foreign investors for contracts or agreements involving contractors, the primary feature of the service contracts
wither technical or financial assistance for large scale was precisely the evil the drafters of the 1987 Constitution
exploration, development and utilization of minerals sought to avoid.
which upon appropriate recommendation of the (DENR) The constitutional provision allowing the President to
Secretary, the President may execute with the foreign enter into FTAAs is an exception to the rule that
proponent. WMCP likewise contended that the annulment participation in the nation‘s natural resources is reserved
of the FTAA would violate a treaty between the exclusively to Filipinos. Accordingly, such provision must
Philippines and Australia which provides for the be construed strictly against their enjoyment by non-
protection of Australian investments. Filipinos. Therefore, RA 7942 is invalid insofar as the
said act authorizes service contracts. Although the statute
ISSUE/S employs the phrase ―financial and
1. Whether or not the Philippine Mining Act is technical agreements in accordance with the 1987
unconstitutional for allowing fully foreign-owned Constitution, its pertinent provisions actually treat
corporations to exploit the Philippine mineral these agreements as service contracts that grant beneficial
resources. ownership to foreign contractors contrary to the
fundamental law.
2. Whether or not the FTAA between the government The underlying assumption in the provisions of the law is
and WMCP is a ―service contract that permits fully that the foreign contractor manages the mineral resources
foreign owned companies to exploit the Philippine just like the foreign contractor in a service contract. By
mineral resources. allowing foreign contractors to manage or operate all the
aspects of the mining operation, RA 7942 has, in effect,
RULING:
conveyed beneficial ownership over the nation‘s mineral
(FIRST ISSUE) RA 7942 or the Philippine Mining Act of resources to these contractors, leaving the State with
1995 is unconstitutional for permitting fully foreign nothing but bare title thereto.
owned corporations to exploit the Philippine natural The same provisions, whether by design or inadvertence,
resources. permit a circumvention of the constitutionally ordained
Article XII Section 2 of the 1987 Constitution retained the 60-40% capitalization requirement for corporations or
associations engaged in the exploitation, development and G.R. NO. 144302
utilization of Philippine natural resources.
May 27,2004
When parts of a statute are so mutually dependent and
connected as conditions, considerations, inducements or
compensations for each other as to warrant a belief that
the legislature intended them as a whole, then if some FACTS: On September 10, 1971, the National Power
parts are unconstitutional, all provisions that are thus Corporation (NPC) entered into a service contract with
dependent, conditional or connected, must fail with them. Philippine Geothermal, Inc. (PGI), a corporation organized
and existing under the laws of California, United States of
Under Article XII Section 2 of the 1987 Charter, foreign
America, for the exploration and exploitation of geothermal
owned corporations are limited only to merely technical
resources covering the Tiwi and Mak-Ban Geothermal
or financial assistance to the State for large scale
Fields. The contract was to expire on 1996 but
exploration, development and utilization of minerals,
negotiations were underway as early as 1994. NPC,
petroleum and other mineral oils. however, was doubtful whether a renewal would be
(SECOND ISSUE) We do not see how a verba legis constitutional in light of Section 2, Article XII of the 1987
approach leads to the conclusion that “the management or Constitution that: The exploration, development, and
utilization of natural resources shall be under the full
operation of mining activities by foreign contractors,
control and supervision of the state.
which is the primary feature of service contracts, was
precisely the evil that the drafters of the 1987 Constitution PGI filed a request for arbitration with the International
sought to eradicate.” Nowhere in the above-quoted Court of Arbitration while NPC filed a petition for
Section can be discerned the objective to keep out of declaratory relief against PGI praying for the determination
foreign hands the management or operation of mining of the constitutionality of the service contract. PGI sought
activities or the plan to eradicate service contracts as these the dismissal of the declaratory relief petition based on
were understood in the 1973 Constitution. Still, petitioners lack of jurisdiction over it in light of the pending arbitration
maintain that the deletion or omission from the 1987 proceedings it instituted but its motion was denied. The
Constitution of the term “service contracts” found in the RTC denied the motion to dismiss.
1973 Constitution sufficiently proves the drafters’ intent to The CA ruled that the petition for declaratory relief should
exclude foreigners from the management of the affected have been dismissed by the RTC as well as by the court
enterprises. To our mind, however, such intent cannot be of appeals in view of the pending arbitration proceedings
definitively and conclusively established from the mere over the same subject matter in view of a breach of the
failure to carry the same expression or term over to the contract subject of the petition.
new Constitution, absent a more specific, explicit and
unequivocal statement to that effect. What petitioners seek During thependency of the case with the SC, PGI and the
(a complete ban on foreign participation in the NPC filed several joint motions to suspend proceedings
management of mining operations, as previously allowed upon the ground that they were negotiating for the
settlement of the case. The motions were granted.
by the earlier Constitutions) is nothing short of bringing
about a momentous sea change in the economic and The parties subsequently filed a Joint Motion to Approve
developmental policies; and the fundamentally capitalist, Compromise Agreement and to Dismiss based on
free-enterprise philosophy of our government. We cannot Compromise Agreement. The compromise agreement
imagine such a radical shift being undertaken by our provided that they have: agreed to terminate the Service
government, to the great prejudice of the mining sector in Contract subject matter of the dispute, in favor of a
particular and our economy in general, merely on the new Geothermal Sales Contract and a PD 1442
basis of the omission of the terms service contract from or Geothermal Service Contract, and PGI has committed
the failure to carry them over to the new Constitution. to form a Philippine company for the development and
There has to be a much more definite and even operation of the Tiwi and Mak-Ban steamfields (Sec. 6.1
unarguable basis for such a drastic reversal of policies. thereof) on a going-forward basis, thereby effectively
erasing any doubt as to the legality of the
compromise.

ISSUE: Whether or not the court can approve the


Compromise Agreement.

PHILIPPINE GEOTHERMAL INC. VS NATIONAL


POWER CORPORATION HELD: No. The assailed decision of the Court of Appeals
dwells on the issue of jurisdiction of the RTC over the
NPC petition for declaratory relief on the constitutionality take actual possession of the land. Ten Forty was not
of the service contract. able to take possession and the SC found it highly
unlikely that they allowed occupation of Marina by
Since only the issue of jurisdiction over the mere tolerance.
constitutionality of a contract was elevated to this Court, it
is beyond its jurisdiction to pass upon and approve the 2. In cases of double sale, the person who first recorded it
Compromise Agreement of the parties. in the Registry of Property shall be considered the
lawful owner. In this case, however, petitioner was
In light of the foregoing development, while this Court unable to establish that the Deed was recorded in the
denies the parties’ Joint Motion to Approve the Registry of Deeds of Olongapo. An unverified notation
Compromise Agreement, it finds the Motion to Dismiss on the Deed is not equivalent to a registration. In the
well-taken. absence of this requirement, the law gives preferential
right to the buyer who in good faith is first in
possession.

3. To determine who is first in possession, the following


TEN FORTY REALTY AND LORENZANA vs CRUZ parameters have been established:
GR NO 151212 SEPTEMBER 10 2013 a. Possession includes not only material but also
symbolic possession
FACTS: b. Possessors in good faith are not aware of any
Ten Forty Realty filed a complaint of ejectment against Marina flaw in their title or mode of acquisition
Cruz who has allegedly occupied the residential lot in Olongapo c. Buyers of property that is in possession of
City, which they bought from Barbara Galino, by virtue of a persons other than the seller must be wary –
Deed of Absolute Sale. It appears that Barbara sold the same lot they must investigate
to Marina who immediately occupied the land. Ten Forty is d. Good faith is always presumed. Burden of
saying the occupation by Marina was merely tolerated by them. proof rests on the one alleging bad faith.
Marina’s defense: (1) Ten Forty, being a corporation, is not Property has not been delivered, hence Ten Forty did not
qualified to own the property which is a public land. (2) Barbara acquire possession either materially or symbolically. Petitioner
Galino did not sell her property to Ten Forty but merely has not proven that respondent was aware of any defect to her
obtained a loan. (3) Ten Forty never occupied the property title. At the time, the property had not been registered which
before she did. Barbara Galino was in possession at the time of was why Marina relied on tax declarations. Galino was actually
the sale, and vacated the lot in favor of Marina. (4) She was the occupying the property when respondent took possession. Thus,
one who caused the cancellation of the tax declaration in the there was no circumstance that could have required her to
name of Barbara and a new one was issued in her name. (5) Ten investigate further.
Forty only obtained its tax declaration 7 months after she did.
MTCC ruled in favor of Ten Forty and ordered Marina to 4. Private corporations are disqualified from acquiring
vacate. lands of public domain. At the time of the sale, there is
RTC reversed. The RTC ruled as follows: 1) respondent’s entry no evidence that the property had already ceased to be
into the property was not by mere tolerance of petitioner, but by of public domain.
virtue of a Waiver and Transfer of Possessory Rights and Deed
of Sale in her favor; 2) the execution of the Deed of Sale
without actual transfer of the physical possession did not have G.R. No. 92024 November 9, 1990
the effect of making petitioner the owner of the property,
because there was no delivery of the object of the sale as CONGRESSMAN ENRIQUE T. GARCIA (Second
provided for in Article 1428 of the Civil Code; and 3) being a District of Bataan), petitioner,
corporation, petitioner was disqualified from acquiring the vs.
property, which was public land. THE BOARD OF INVESTMENTS, THE DEPARTMENT
CA affirmed: Case cannot be unlawful detainer because there OF TRADE AND INDUSTRY, LUZON
has been no prior contract between the parties. Neither can it be PETROCHEMICAL CORPORATION, and PILIPINAS
forcibly entry because there is no showing that there was prior SHELL CORPORATION, respondents.
physical possession by the petitioner.
GUTIERREZ, JR., J.: This is a petition to annul and set
ISSUE: WON Marina may be validly ejected from the property
aside the decision of the Board of Investments
RULING: No. (BOI)/Department of Trade and Industry (DTI) approving
1. In a contract of sale, the buyer acquires the thing sold the transfer of the site of the proposed petrochemical
only upon its delivery. The execution of a public plant from Bataan to Batangas and the shift of feedstock
instrument gives rise to a presumption of delivery, but
this presumption is destroyed when delivery is not for that plant from naphtha only to naphtha and/or
effected because of a legal impediment. Constructive liquefied petroleum gas (LPG).
delivery is deemed negated upon failure of vendee to
FACTS: Former Bataan Petrochemical Corporation (BPC),
now Luzon Petrochemical Corporation, formed by a original application of the BPC to have its plant site in
group of Taiwanese investors, was granted by the BOI for Bataan and the product naphta as feedstock maintained.
the transfer of its proposed plant site from Bataan to
First, Bataan was the original choice as the plant site of
Batangas and the shift of the plant’s feedstock or fuel for the BOI to which the BPC agreed. That is why it organized
its petrochemical plant from “naphta only” to “naptha itself into a corporation bearing the name Bataan. There is
and/or liquefied petroleum gas. The reason adduced for available 576 hectares of public land precisely reserved as
the transfer was the insurgency and unstable labor the petrochemical zone in Limay, Bataan under P.D. No.
situation, and the presence in Batangas of a huge 1803. There is no need to buy expensive real estate for
the site unlike in the proposed transfer to Batangas. The
liquefied petroleum gas (LPG) depot owned by the
site is the result of careful study long before any covetous
Philippine Shell Corporation. In February 1989, one year interests intruded into the choice. The site is ideal. It is not
after the BPC began its production in Bataan, the unduly constricted and allows for expansion. The
corporation applied to the BOI to have its plant site respondents have not shown nor reiterated that the
transferred from Bataan to Batangas. Despite vigorous alleged peace and order situation in Bataan or unstable
opposition from petitioner Cong. Enrique Garcia and labor situation warrant a transfer of the plant site to
Batangas. Certainly, these were taken into account when
others, the BOI granted private respondent BPC’s
the firm named itself Bataan Petrochemical Corporation.
application, stating that the investors have the final Moreover, the evidence proves the contrary.
choice as to where to have their plant site because they
are the ones who risk capital for the project. Second, the BRC, a government
owned Filipino corporation, located in Bataan produces
60% of the national output of naphtha which can be used
ISSUE: as feedstock for the plant in Bataan. It can provide the
1. Whether or not the foreign investor has the right of final feedstock requirement of the plant. On the other hand, the
choice of plant site. country is short of LPG and there is need to import the
same for use of the plant in Batangas. The local
2. Whether it constitutes a grave abuse of discretion for production thereof by Shell can hardly supply the needs of
the BOI to yield to the wishes of the investor without the consumers for cooking purposes. Scarce dollars will
considering the national interest be diverted, unnecessarily, from vitally essential projects
in order to feed the furnaces of the transferred
Ruling: petrochemical plant.
1.Does the investor have a "right of final choice" of plant
Third, naphtha as feedstock has been exempted by law
site? Neither under the 1987 Constitution nor in the
from the ad valorem  tax by the approval of Republic Act
Omnibus Investments Code is there such a 'right
No. 6767 by President Aquino but excluding LPG from
of final choice.' In the first place, the investor's choice is exemption from ad valorem tax. The law was enacted
subject to processing and approval or disapproval by the specifically for the petrochemical industry. The policy
BOI (Art. 7, Chapter II, Omnibus Investments Code). By determination by both Congress and the President is
submitting its application and amended application to the clear. Neither BOI nor a foreign investor should disregard
BOI for approval, the investor recognizes the sovereign or contravene expressed policy by shifting the feedstock
prerogative of our Government, through the BOI, to from naphtha to LPG.
approve or disapprove the same after determining
whether its proposed project will be feasible, desirable and Fourth, under Section 10, Article XII of the 1987
beneficial to our country. By asking that his opposition to Constitution, it is the duty of the State to "regulate and
the LPC's amended application be heard by the BOI, the exercise authority over foreign investments within its
petitioner likewise acknowledges that the BOI, not the national jurisdiction and in accordance with its national
investor, has the last word or the "final choice" on the goals and priorities." The development of a self-reliant and
matter. independent national economy effectively controlled by
Filipinos is mandated in Section 19, Article II of the
Secondly, as this case has shown, even a choice that had Constitution.
been approved by the BOI may not be 'final', for
supervening circumstances and changes in the conditions In Article 2 of the Omnibus Investments Code of 1987 "the
of a place may dictate a corresponding change in the sound development of the national economy in
choice of plant site in order that the project will not fail. consonance with the principles and objectives of
After all, our country will benefit only when a project economic nationalism" is the set goal of government.
succeeds, not when it fails
Fifth, with the admitted fact that the investor is raising the
2. The Supreme Court found the BOI to have committed greater portion of the capital for the project from local
grave abuse of discretion in this case and ordered the sources by way of loan which led to the so-called
"petroscam scandal", the capital requirements would be MANILA PRINCE HOTEL petitioner,
greatly minimized if LPC does not have to buy the land for vs.
the project and its feedstock shall be limited to naphtha GOVERNMENT SERVICE INSURANCE SYSTEM,
which is certainly more economical, more readily available MANILA HOTEL CORPORATION, COMMITTEE ON
than LPG, and does not have to be imported. PRIVATIZATION and OFFICE OF THE
GOVERNMENT CORPORATE COUNSEL,
Sixth, if the plant site is maintained in Bataan, the PNOC respondents.
shall be a partner in the venture to the great benefit and
advantage of the government which shall have a Pertinent Topic: The Filipino First Policy as
participation in the management of the project instead of a provided for under Sec. 10, second par., Art. XII,
firm which is a huge multinational corporation. of the 1987

In the light of all the clear advantages manifest in the Facts:


plant's remaining in Bataan, practically nothing is shown to
justify the transfer to Batangas except a near-absolute The controversy arose when respondent Government
discretion given by BOI to investors not only to freely Service Insurance System (GSIS), pursuant to the
choose the site but to transfer it from their own first choice privatization program of the Philippine Government,
for reasons which remain murky to say the least. decided to sell through public bidding 30% to 51% of
the issued and outstanding shares of respondent
And this brings us to a prime consideration which the Manila Hotel Corporation (MHC). The winning
Court cannot rightly ignore. bidder, or the eventual “strategic partner,” will
provide management expertise or an international
marketing/reservation system, and financial support
Section 1, Article XII of the Constitution provides that:
to strengthen the profitability and performance of
the Manila Hotel.
x x x           x x x          x x x
In a close bidding held on 18 September 1995 only
The State shall promote industrialization two (2) bidders participated: petitioner Manila Prince
and full employment based on sound Hotel Corporation, a Filipino corporation, which
agricultural development and agrarian offered to buy 51% of the MHC or 15,300,000 shares
reform, through industries that make full at P41.58 per share, and Renong Berhad, a
and efficient use of human and natural Malaysian firm, with ITT-Sheraton as its hotel
resources, and which are competitive in operator, which bid for the same number of shares
both domestic and foreign markets. at P44.00 per share, or P2.42 more than the bid of
However, the State shall protect Filipino petitioner. Prior to the declaration of Renong
enterprises against unfair foreign Berhard as the winning bidder, petitioner Manila
competition and trade practices. Prince Hotel matched the bid price and sent a
manager’s check as bid security, which GSIS refused
xxx xxx xxx to accept.

Apprehensive that GSIS has disregarded the tender


Every provision of the Constitution on the national of the matching bid and that the sale may be
economy and patrimony is infused with the spirit of consummated with Renong Berhad, petitioner filed a
national interest. The non-alienation of natural resources,
petition before the Court.
the State's full control over the development and utilization
of our scarce resources, agreements with foreigners being  
based on real contributions to the economic growth and
general welfare of the country and the regulation of foreign Issues:
investments in accordance with national goals and
priorities are too explicit not to be noticed and understood.
1. Whether or not Sec. 10, second par., Art. XII,
of the 1987 Constitution is a self-executing
provision.
2. Whether or not the Manila Hotel forms part
of the national patrimony.
3. Whether or not the submission of matching
bid is premature
MANILA PRINCE HOTEL VS. GSIS 4. Whether or not there was grave abuse of
G.R. NO. 122156. February 3, 1997 discretion on the part of the respondents in refusing
the matching bid of the petitioner. national economy and patrimony, the state shall give

Rulings: preference to qualified Filipinos--qualified Filipinos

In the resolution of the case, the Court held that: shall be preferred.

1. Yes, it is a self-executing provision. "Qualified Filipinos" includes corporations at least


60% of which is Filipino owned. This means that
Article XII, Sec. 10 provides:
preference shall be given to those citizens who can
Section 10. The Congress shall, upon make a viable contribution to the common good,
recommendation of the economic and planning
agency, when the national interest dictates, reserve because of credible competence and efficiency.
to citizens of the Philippines or to corporations or
associations at least sixty per centum of whose
capital is owned by such citizens, or such higher National Patrimony refers not only to natural
percentage as Congress may prescribe, certain areas
resources of the Philippines but also cultural
of investments. The Congress shall enact measures
that will encourage the formation and operation of heritage.
enterprises whose capital is wholly owned by
Filipinos.
Manila Hotel has become a living testimonial of
In the grant of rights, privileges, and
concessions covering the national economy and Philippine Heritage, it is dubbed as the Official
patrimony, the State shall give preference to Guest House of the Philippine Government. Its
qualified Filipinos.
existence is impressed with public interest.
The State shall regulate and exercise authority over
foreign investments within its national jurisdiction
and in accordance with its national goals and
priorities.

Since the Constitution is the fundamental,


paramount and supreme law of the nation, it is A constitutional provision is self-executing if the

deemed written in every statute and contract. A nature and extent of the right conferred and the

provision which lays down a general principle, such liability imposed are fixed by the constitution itself,
as those found in Art. II of the 1987 Constitution, is so that they can be determined by an examination

usually not self-executing. But a provision which is and construction of its terms, and there is no

complete in itself and becomes operative without language indicating that the subject is referred to

the aid of supplementary or enabling legislation, the legislature for action. Unless it is expressly

or that which supplies sufficient rule by means of provided that a legislative act is necessary to enforce

which the right it grants may be enjoyed or a constitutional mandate, the presumption now is

protected, is self-executing. that all provisions of the constitution are self-


executing. If the constitutional provisions are treated

Sec.10 Par.2 Art. 12, is a provision complete in as requiring legislation instead of self-executing, the
legislature would have the power to ignore and
itself and needs no further guidelines or
practically nullify the mandate of the fundamental
implementing laws or rules for its enforcement.
law.

When our Constitution mandates that in the grant 10, second par., Art. XII of the 1987 Constitution is
of rights privileges and concessions covering a mandatory, positive command which is complete
in itself and which needs no further guidelines or of the MHC comes within the purview of the
implementing laws or rules for its enforcement. constitutional shelter for it comprises the majority
From its very words the provision does not require and controlling stock, so that anyone who acquires
any legislation to put it in operation. It is per or owns the 51% will have actual control and
se judicially enforceable. When our Constitution management of the hotel. In this instance, 51% of
mandates that in the grant of rights, privileges, and the MHC cannot be disassociated from the hotel and
concessions covering national economy and the land on which the hotel edifice stands.
patrimony, the State shall give preference to
qualified Filipinos, it means just that – qualified
Filipinos shall be preferred. And when our
Constitution declares that a right exists in certain 3. It is not premature.

specified circumstances an action may be


In the instant case, where a foreign firm submits the
maintained to enforce such right notwithstanding
highest bid in a public bidding concerning the grant
the absence of any legislation on the subject;
of rights, privileges and concessions covering the
consequently, if there is no statute especially
national economy and patrimony, thereby exceeding
enacted to enforce such constitutional right, such
the bid of a Filipino, there is no question that the
right enforces itself by its own inherent potency and
Filipino will have to be allowed to match the bid of
puissance, and from which all legislations must take
the foreign entity. And if the Filipino matches the bid
their bearings. Where there is a right there is a
of a foreign firm the award should go to the Filipino.
remedy. Ubi jus ibi remedium.
It must be so if the Court is to give life and meaning
to the Filipino First Policy provision of the 1987
Constitution. For, while this may neither be
2. Yes expressly stated nor contemplated in the bidding
rules, the constitutional fiat is omnipresent to be
In its plain and ordinary meaning, the term simply disregarded. To ignore it would be to sanction
patrimony pertains to heritage. When the a perilous skirting of the basic law.
Constitution speaks of national patrimony, it refers
not only to the natural resources of the Philippines, The Court does not discount the apprehension that
as the Constitution could have very well used the this policy may discourage foreign investors. But the
term natural resources, but also to the cultural Constitution and laws of the Philippines are
heritage of the Filipinos. understood to be always open to public scrutiny.
These are given factors which investors must
It also refers to Filipino’s intelligence in arts, consider when venturing into business in a foreign
sciences and letters. In the present case, Manila jurisdiction. Any person therefore desiring to do
Hotel has become a landmark, a living testimonial of business in the Philippines or with any of its
Philippine heritage. While it was restrictively an agencies or instrumentalities is presumed to know
American hotel when it first opened in 1912, a his rights and obligations under the Constitution
concourse for the elite, it has since then become the and the laws of the forum.
venue of various significant events which have
shaped Philippine history.

Verily, Manila Hotel has become part of our national 4. There was grave abuse of discretion.
economy and patrimony. For sure, 51% of the equity
To insist on selling the Manila Hotel to foreigners Petitioners: Wigberto Tanada, et al.

when there is a Filipino group willing to match the Respondents: Edgardo Angara, et al.
bid of the foreign group is to insist that government Ponente: Panganiban, J.
be treated as any other ordinary market player, and
bound by its mistakes or gross errors of judgement,
SUMMARY:
regardless of the consequences to the Filipino
people. The miscomprehension of the Constitution is Petitioners assail the constitutionality of the Philippines
acceding to the World Trade Organization for being violative of
regrettable. Thus, the Court would rather remedy
provisions which are supposed to give preference to Filipino
the indiscretion while there is still an opportunity to workers and economy and on the ground that it infringes
do so than let the government develop the habit of legislative and judicial power. The WTO, through it provisions
on “most favored nation” and national treatment, require that
forgetting that the Constitution lays down the basic
nationals and other member countries are placed in the same
conditions and parameters for its actions. footing in terms of products and services. However, the Court
brushed off these contentions and ruled that the WTO is
Since petitioner has already matched the bid price constitutional. Sections 10 and 12 of Article XII (National
Economy and Patrimony) should be read in relation to Sections
tendered by Renong Berhad pursuant to the bidding
1 and 13 (promoting the general welfare). Also, Section 10 is
rules, respondent GSIS is left with no alternative but self-executing only to “rights, privileges, and concessions
to award to petitioner the block of shares of MHC covering national economy and patrimony” but not every aspect
of trade and commerce. There are balancing provisions in the
and to execute the necessary agreements and
Constitution allowing the Senate to ratify the WTO agreement.
documents to effect the sale in accordance not only Also, the Constitution doesn’t rule out foreign competition.
with the bidding guidelines and procedures but with States waive certain amount of sovereignty when entering into
treaties.
the Constitution as well. The refusal of respondent
GSIS to execute the corresponding documents with
FACTS:
petitioner as provided in the bidding rules after the
latter has matched the bid of the Malaysian firm The Philippines joined World Trade Organization as a
founding member with the goal of improving Philippine access
clearly constitutes grave abuse of discretion.
to foreign markets, especially its major trading partners, through
the reduction of tariffs on its exports. The President also saw in
  the WTO the opening of new opportunities for the services
Hence, respondents GOVERNMENT SERVICE sector, the reduction of costs and uncertainty associated with
exporting and the attraction of more investments into the
INSURANCE SYSTEM, MANILA HOTEL
country. On April 15, 1994, respondent Navarro, then DTI
CORPORATION, COMMITTEE ON PRIVATIZATION
Secretary, signed in Marrakesh, Morocco, the Final Act
and OFFICE OF THE GOVERNMENT CORPORATE
Embodying the Results of the Uruguay Round of Multilateral
COUNSEL are directed to CEASE and DESIST from
Negotiations. On December 14, 1994, the Senate concurred in
selling 51% of the shares of the Manila Hotel
the ratification of the President of the Philippines of the
Corporation to RENONG BERHAD, and to ACCEPT
Agreement Establishing the WTO which includes various
the matching bid of petitioner MANILA PRINCE
agreements and associated legal instruments. On December 16,
HOTEL CORPORATION to purchase the subject 51%
1994, the President signed the Instrument of Ratification.
of the shares of the Manila Hotel Corporation
at P44.00 per share and thereafter to execute the
necessary agreements and documents to effect the
sale, to issue the necessary clearances and to do
ISSUES:
such other acts and deeds as may be necessary for
the purpose.
1. Whether the WTO Agreement violated the mandated
economic nationalism by the Constitution

2. Whether the provisions of the WTO Agreement restrict and


impairs Philippine sovereignty, specifically the legislative
power vested in the Congress

Tañada v. Angara G.R. No. 118295 | May 2, 1997 3. Whether the Senate concurrence in the WTO Agreement and
its annexes but not in the other documents referred to in the
Final Act is defective and insufficient and thus constitutes abuse FRANCISCO S. TATAD, JOHN H. OSMEÑA and
of discretion RODOLFO G. BIAZON, petitioners, vs. HON.
JESUS B. GARCIA, JR., in his capacity as the
Secretary of the Department of
Transportation and Communications, and
RULING:
EDSA LRT CORPORATION, LTD., respondents.
1. No. The Constitution did not intend to pursue an isolationist QUIASON, J.:
policy. It did not shut out foreign investments, goods and
services in the development of the Philippine economy. In fact, Petitioners Francisco S. Tatad, John H. Osmena
it allows an exchange on the basis of equality and reciprocity,
and Rodolfo G. Biazon are are suing in their
frowning only on foreign competition that is unfair. The
constitutional policy of a self-reliant and independent national capacities as Senators and as taxpayers.
economy does not necessarily rule out the entry of foreign Respondent Secretary of the Department of
investments, goods and services. It contemplates neither Transportation and Communications (DOTC), while
economic seclusion nor mendicancy in the international private respondent EDSA LRT Corporation, Ltd. is a
community.
private corporation organized under the laws of
2. No. While sovereignty has traditionally been deemed absolute Hongkong.
and all-encompassing on the domestic level, it is however
subject to restrictions and limitations voluntarily agreed to by FACTS
the Philippines, expressly or impliedly, as a member of the
In 1989, DOTC planned to construct a light railway
family of nations. Unquestionably, the Constitution did not
envision a hermit-type isolation of the country from the rest of transit line along EDSA, referred to as EDSA Light
the world. By the doctrine of incorporation, the country is bound Rail Transit III (EDSA LRT III), was intended to
by generally accepted principles of international law, which are provide a mass transit system along EDSA and
considered to be automatically part of our laws. A treaty alleviate the congestion and growing
engagement is not a mere moral obligation on the parties. By
their inherent nature, treaties really limit or restrict the transportation problem in the metropolis.
absoluteness of sovereignty. The Philippines has effectively
R.A. No. 6957 entitled "An Act Authorizing the
agreed to limit the exercise of its sovereign powers of taxation,
eminent domain and police power. The underlying consideration Financing, Construction, Operation and
in this partial sovereignty is the reciprocal commitment of the Maintenance of Infrastructure Projects by the
other contracting states in granting the same privilege and Private Sector, and For Other Purposes," was and
immunities to the Philippines, its officials and its citizens. The referred to as the Build-Operate-Transfer
same reciprocity characterizes the same commitments under
(BOT) Law.
WTO-GATT. The point is that a portion of sovereignty may be
waived without violating the Constitution, based on the rationale BOT Law provides for two schemes for the
that the Philippines adopts the generally accepted principles of
international law as part of the law of the land and adheres to the financing, construction and operation of
policy of cooperation and amity with all nations. government projects through private initiative and
investment: Build-Operate-Transfer (BOT) or
Build-Transfer (BT).
3. No. The petitioners submit that concurrence in the WTO
Agreement alone is flawed because it is in effect a rejection of The Prequalification Bids and Awards Committee
the Final Act. The Court held that a final act is an instrument (PBAC) and the Technical Committee] issued
which records the winding up of the proceedings of a diplomatic
conference and not the treaty itself. On the other hand, the WTO guidelines for the prequalification of contractors for
Agreement itself expresses what multilateral agreements are the financing and implementation of the project.
deemed included as its integral parts. It should be added that the Five groups responded to the invitation namely:
Senate was well-aware of what it was concurring in as shown by
the member’s deliberation. ABB Trazione of Italy,
Hopewell Holdings Ltd. of Hongkong,
Mansteel International of Mandaue, Cebu,
Mitsui & Co., Ltd. of Japan, and
EDSA LRT Consortium, composed of ten foreign
and domestic corporations

After evaluating the prequalification, only the


EDSA LRT Consortium "met the requirements.
Finding the proposal by EDSA to be in compliance (1) WON the EDSA LRT III is a public utility, and
with the bid requirements, DOTC and respondent the ownership and operation thereof is limited by
EDSA LRT Corporation, Ltd., in substitution of the the Constitution to Filipino citizens and domestic
EDSA LRT Consortium, entered into an "Agreement corporations, not foreign corporations like private
to Build, Lease and Transfer a Light Rail Transit respondent;
System for EDSA" under the terms of the BOT Law
(2) WON the Build-Lease-Transfer (BLT) scheme
and thereafter requested presidential approval of
provided in the agreements is not the BOT or BT
the contract.
Scheme under the law;
Executive Secretary Franklin Drilon informed
(3) the contract to construct the EDSA LRT III was
Secretary [DOTC] that the President could not
awarded to private respondent not through public
grant the requested approval because DOTC failed
bidding which is the only mode of awarding
to conduct actual public bidding and that the public
infrastructure projects under the BOT law; and
bidding is the only mode to award BOT projects,
and the prequalification proceedings was not the HELD
public bidding contemplated under the law.
1. [No].
In view of this, DOTC and private respondents re-
negotiated the agreement. The parties entered into Private respondent EDSA LRT Corporation, Ltd. to
a "Revised and Restated Agreement to Build, whom the contract to construct the EDSA LRT III
Lease and Transfer a Light Rail Transit System for was awarded by public respondent, is admittedly a
EDSA" so as to "clarify their respective rights and foreign corporation "duly incorporated and existing
responsibilities" and to submit it to the President. under the laws of Hongkong".
Such was approved in a Memorandum. Section 11 of Article XII of the Constitution
According to the agreements, private provides:
respondents shall undertake and finance the No franchise, certificate or any
entire project required. Upon full or partial other form of authorization for
completion and viability thereof, private the operation of a public
respondent shall deliver the use and utility shall be granted except to
possession of the completed portion to DOTC citizens of the Philippines or to
which shall operate the same. DOTC shall pay corporations or associations
private respondent rentals on a monthly basis organized under the laws of the
through an Irrevocable Letter of Credit. As Philippines at least sixty per
agreed upon, private respondent's capital centum of whose capital is
shall be recovered from the rentals to be paid owned by such citizens, nor shall
by the DOTC which, in turn, shall come from such franchise, certificate or
the earnings of the EDSA LRT III. After 25 authorization be exclusive
years and DOTC shall have completed character or for a longer period
payment of the rentals, ownership of the than fifty years . . . (Emphasis
project shall be transferred to the latter for a supplied).
consideration of only U.S. $1.00.
The Constitution, in no uncertain terms, requires a
Subsequently, R.A. No. 7718, an "Act Amending franchise for the operation of a public utility.
Certain Sections of Republic Act No. 6957, Entitled However, it does not require a franchise before
"An Act Authorizing the Financing, Construction, one can own the facilities needed to operate a
Operation and Maintenance of Infrastructure public utility so long as it does not operate them to
Projects by the Private Sector, and for Other serve the public.
Purposes" was signed. The law expressly
recognizes BLT scheme and allows direct Even the mere formation of a public utility
negotiation of BLT contracts. corporation does not ipso facto characterize the
corporation as one operating a public utility. The
ISSUES moment for determining the requisite Filipino
nationality is when the entity applies for a assume all the obligations and liabilities of a
franchise, certificate or any other form of common carrier. In sum, private respondent
authorization for that purpose. will not run the light rail vehicles and collect
fees from the riding public. It will have no
In law, there is a clear distinction between the
dealings with the public and the public will
"operation" of a public utility and the ownership of
have no right to demand any services from it.
the facilities and equipment used to serve the
public. 2. [No]

The exercise of the rights encompassed in The BOT scheme is expressly defined as one where
ownership is limited by law so that a property the contractor undertakes the construction and
cannot be operated and used to serve the public as financing in infrastructure facility, and operates
a public utility unless the operator has a franchise. and maintains the same. The contractor operates
the facility for a fixed period during which it may
The right to operate a public utility may exist
recover its expenses and investment in the project
independently and separately from the ownership
plus a reasonable rate of return thereon. After the
of the facilities thereof. One can own said facilities
expiration of the agreed term, the contractor
without operating them as a public utility, or
transfers the ownership and operation of the
conversely, one may operate a public utility
project to the government.
without owning the facilities used to serve the
public. The devotion of property to serve the public Emphasis must be made that under the BOT
may be done by the owner or by the person in scheme, the owner of the infrastructure facility
control thereof who may not necessarily be the must comply with the citizenship requirement of
owner thereof. the Constitution on the operation of a public utility.
No such a requirement is imposed in the BT
What private respondent owns are the rail tracks,
scheme.
rolling stocks like the coaches, rail stations,
terminals and the power plant, not a public utility. In the BT scheme, the contractor undertakes the
While a franchise is needed to operate these construction and financing of the facility, but after
facilities to serve the public, they do not by completion, the ownership and operation thereof
themselves constitute a public utility. What are turned over to the government. The
constitutes a public utility is not their ownership government, in turn, shall pay the contractor its
but their use to serve the public. total investment on the project in addition to a
reasonable rate of return. If payment is to be
While private respondent is the owner of the
effected through amortization payments by the
facilities necessary to operate the EDSA. LRT III, it
government infrastructure agency or local
admits that it is not enfranchised to operate a
government unit concerned, this shall be made in
public utility. In view of this incapacity, private
accordance with a scheme proposed in the bid and
respondent and DOTC agreed that on completion
incorporated in the contract (R.A. No. 6957, Sec.
date, private respondent will immediately deliver
6). The BLT scheme in the challenged
possession of the LRT system by way of lease for
agreements is but a variation of the BT
25 years, during which period DOTC shall operate
scheme under the law.
the same as a common carrier and private
respondent shall provide technical maintenance There is no mention in the BOT Law that the BOT
and repair services to DOTC. and BT schemes bar any other arrangement for
the payment by the government of the project
Fees for private respondent' s services shall be
cost. As a matter of fact, the burden on the
included in the rent, which likewise includes the
government in raising funds to pay for the project
project cost, cost of replacement of plant
is made lighter by allowing it to amortize payments
equipment and spare parts, investment and
out of the income from the operation of the LRT
financing cost, plus a reasonable rate of return
System.
thereon.
3. [No]
Since DOTC shall operate the EDSA LRT III, it shall
The fact that the contract for the construction of prequalification requirements, after
the EDSA LRT III was awarded through negotiation which it is required to submit a bid
does not suffice to invalidate the award. proposal . . .

The records show that only one applicant passed Petitioners' claim that the BLT scheme and direct
the prequalification process. Since only one was negotiation of contracts are not contemplated by
left, to conduct a public bidding in accordance with the BOT Law has now been rendered moot and
Section 5 of the BOT Law for that lone participant academic by R.A. No. 7718. Section 3 of this law
will be an absurb and pointless exercise. authorizes all government infrastructure agencies,
government-owned and controlled corporations
Contrary to the comments of the Executive
and local government units to enter into contract
Secretary Drilon, Section 5 of the BOT Law in
with any duly prequalified proponent for the
relation to Presidential Decree No. 1594 allows the
financing, construction, operation and maintenance
negotiated award of government infrastructure
of any financially viable infrastructure or
projects.
development facility through a BOT, BT, BLT, BOO
Indeed, where there is a lack of qualified bidders (Build-own-and-operate), CAO (Contract-add-
or contractors, the award of government operate), DOT (Develop-operate-and-transfer),
infrastructure contracts may be made by ROT (Rehabilitate-operate-and-transfer), and ROO
negotiation. Presidential Decree No. 1594 is the (Rehabilitate-own-operate) (R.A. No. 7718, Sec. 2
general law on government infrastructure [b-j]).
contracts while the BOT Law governs particular
4. [No]
arrangements or schemes aimed at encouraging
private sector participation in government It must be noted that as part of the EDSA LRT III
infrastructure projects. The two laws are not project, private respondent has been granted, for
inconsistent with each other but are in pari a period of 25 years, exclusive rights over the
materia and should be read together accordingly. depot and the air space above the stations for
development into commercial premises for lease,
There is nothing in our laws that prohibits parties
sublease, transfer, or advertising. For and in
to a contract from renegotiating and modifying in
consideration of these development rights,
good faith the terms and conditions thereof so as
private respondent shall pay DOTC in
to meet legal, statutory and constitutional
Philippine currency guaranteed revenues
requirements. Under the circumstances, to require
generated therefrom in the amounts set forth
the parties to go back to step one of the
in the Supplemental Agreement. In the event
prequalification process would just be an idle
that DOTC shall be unable to collect the
ceremony.
guaranteed revenues, DOTC shall be allowed to
Republic Act No. 7718 is intended to provide deduct any shortfalls from the monthly rent due
financial incentives and "a climate of minimum private respondent for the construction of the
government regulations and procedures and EDSA LRT III. All rights, titles, interests and
specific government undertakings in support of the income over all contracts on the commercial
private sector" (Sec. 1). Section 5-A of R.A. No. spaces shall revert to DOTC upon expiration of the
7718, which expressly allows direct negotiation of 25-year period.
contracts, provides:
That the grantee of a government contract will
Direct Negotiation of Contracts. — profit therefrom and to that extent the government
Direct negotiation shall be is deprived of the profits if it engages in the
resorted to when there is only business itself, is not worthy of being raised as an
one complying bidder left as issue. In all cases where a party enters into a
defined hereunder. contract with the government, he does so, not out
of charity and not to lose money, but to gain
(a) If, after advertisement, only one pecuniarily.
contractor applies for
prequalification and it meets the
ASSOCIATED COMMUNICATIONS & WIRELESS SERVICES- shall be authorized to operate without the
UNITED BROADCASTING NETWORKS, petitioner, vs. authority of the Board of Communications and
NATIONAL TELECOMMUNICATIONS COMMISSION, the Secretary of Public Works and
respondent. Communications or their successors who have
February 17, 2003 the right and authority to assign to qualified
parties frequencies, channels or other means of
identifying broadcasting system; Provided,
Facts: however, that any conflict over, or
disagreement with a decision of the
On November 11, 1931, Act No. 3846, entitled “An Act aforementioned authorities may be appealed
Providing for the Regulation of Radio Stations and Radio finally to the Office of the President within
Communications in the Philippines and for Other Purposes,” fifteen days from the date the decision is
was enacted. Sec. 1 of the law reads, viz.: received by the party in interest.”

“Sec. 1. No person, firm, company, A few years later or in 1979, E.O. No. 5464 was issued. It
association, or corporation shall construct, integrated the Board of Communications and the
install, establish, or operate a radio Telecommunications Control Bureau under the Integrated
transmitting station, or a radio receiving Reorganization Plan of 1972 into the NTC. Among the
station used for commercial purposes, or a powers vested in the NTC under Sec. 15 of E.O. No. 546 are
radio broadcasting station, without having the following:
first obtained a franchise therefor from the
Congress of the Philippines . . .” “a. Issue Certificate of Public
Convenience for the operation of
Pursuant to the above provision, Congress enacted in 1965 communication utilities and services,
R.A. No. 4551, entitled “An Act Granting Marcos J. Villaverde, radio communications systems, wire or
Jr. and Winfred E. Villaverde a Franchise to Construct, wireless telephone or telegraph system,
Install, Maintain and Operate Public Radiotelephone and radio and television broadcasting
Radiotelegraph Coastal Stations, and Public Fixed and Public system and other similar public utilities;
Based and Land Mobile Stations within the Philippines for
the Reception and Transmission of Radiotelephone and x x x     x x x     x x x
Radiotelegraph for Domestic Communications and
Provincial Telephone Systems in Certain Provinces.” It gave c. Grant permits for the use of radio
the grantees a 50-year franchise. In 1969, the franchise was frequencies for wireless telephone and
transferred to petitioner Associated Communications & telegraph systems and radio
Wireless Services-United Broadcasting Network, Inc. (ACWS communication systems including
for brevity) through Congress’ Concurrent Resolution No. amateur radio stations and radio and
58.3 Petitioner ACWS then engaged in the installation and television broadcasting systems; . . .”
operation of several radio stations around the country.
Upon termination of petitioner’s franchise on December 31,
In 1974, P.D. No. 576-A, “Regulating the Ownership and 1981 pursuant to P.D. No. 576-A, it continued operating its
Operation of Radio and Television Stations and for other radio stations under permits granted by the NIC.
Purposes” was issued, with the following pertinent
provisions on franchise of radio and television
broadcasting systems: Issue
Whether or not the operation of a radio or television station
“Sec. 1. No radio station or television channel requires a congressional franchise.
may obtain a franchise unless it has sufficient
capital on the basis of equity for its operation
for at least one year, including purchase of
equipment. Ruling:
Yes.
x x x     x x x     x x x
The appellate court correctly ruled that a
Sec. 6. All franchises, grants, licenses, permits, congressional franchise is necessary for petitioner to
certificates or other forms of authority to operate television Channel 25. Even assuming that Act No.
operate radio or television broadcasting 3846 applies only to radio stations and not to television
systems shall terminate on December 31, 1981. stations as petitioner adamantly insists, the subsequent P.D.
Thereafter, irrespective of any franchise, grant, No. 576-A clearly shows in Section 1 that a franchise is
license, permit, certificate or other forms of required to operate radio as well as television stations, viz.:
authority to operate granted by any office, “Sec. 1. No radio station or television channel may obtain a
agency or person, no radio or television station franchise unless it has sufficient capital on the basis of
equity for its operation for at least one year, including the prohibitions shall, after notice and hearing, be banned
purchase of equipment.” permanently from participating. All insurance companies who
are blacklisted in any government agency or instrumentality
As pointed out in DOJ Opinion No. 98, there is
nothing in P.D. No. 576-A that reveals any intention to do including court and other quasi-judicial agencies are
away with the requirement of a franchise for the operation automatically disallowed to participate in this program.
of radio and television stations. Section 6 of P.D. No. 576-A Accordingly, no policy or certificate of cover shall be accepted
merely identifies the regulatory agencies from whom from these companies as proof of compliance with this
authorizations, in addition to the required congressional program. The Board shall issue from time to time the list of the
franchise, must be secured after December 31, 1981, viz.: blacklisted or suspended companies.
“Sec. 6. All franchises, grants, licenses, permits, certificates
or other forms of authority to operate radio or television The LTFRB made a one month nationwide information
broadcasting systems shall terminate on December 31, campaign on the nature of the two-group system and of the
1981. Thereafter, irrespective of any franchise, grant, blacklisting scheme and in a meeting with the different
license, permit, certificate or other forms of authority to
insurance companies, including EASCO was blacklisted by the
operate granted by any office, agency or person, no radio or
RTC QC. Thus prompted them to file a Petition
television station shall be authorized to operate without the
authority of the Board of Communications and the Secretary for Certiorari and Prohibition with the CA questioning the
of Public Works and Communications or their successors validity of those issuances.
who have the right and authority to assign to qualified
parties frequencies, channels or other means of identifying Issue: Won Petitioner the subsequent implementing Circulars
broadcasting system . . .” violate the constitutional proscription against monopoly as well
as unfair competition and combination in restraint of trade

Held: The constitutional provision on monopolies is found in


Article XII as follows:
EASTERN ASSURANCE & SURETY CORPORATION (EASCO), Sec. 19. The State shall regulate or prohibit monopolies when
Petitioner, vs. LAND TRANSPORTATION FRANCHISING and the public interest so requires. No combinations in restraint of
REGULATORY BOARD (LTFRB), respondent.\ trade or unfair competition shall be allowed.
PANGANIBAN, J.: While embracing free enterprise as an economic creed, the
The Facts Constitution does not totally prohibit the operation of
monopolies.10 However, it mandates the State to regulate them
LTFRB conducted a thorough investigation on the sufficiency of when public interest so requires.
existing insurance policies for PUVs. In the course of its
investigation, the Board discovered that insurance coverage of Undoubtedly, Memorandum Circular No. 2001-010 authorized
PUVs was only P50, 000.00 for the entire vehicle regardless of and regulated two separate monopolies. In Garcia v.
the number of passengers or persons killed or injured. Corona,12  the Court stated:
Thereafter, the Board issued Memorandum Circular fixing the The simplest form of monopoly exists when there is only one
insurance coverage of PUVs on the basis of the number of seller or producer of a product or service for which there are no
persons that may be killed or injured instead of the entire substitute. In its more complex form, monopoly is defined as
vehicle alone. the joint acquisition or maintenance by members of a
After a year of implementation, the Board now has received conspiracy formed for that purpose, of the power to control
numerous complaints. So the board made several amendments and dominate trade and commerce in a commodity to such an
In order to make sure that future claims of PUV operators and extent that they are able, as a group, to exclude actual or
passenger accident victims are paid within the required time, potential competitors from the field, accompanied with the
and in order to minimize, if not eliminate, fake certificates of intention or purpose to exercise such power.
cover and graft and corruption, as well as to ensure the Indeed, in authorizing and regulating the two insurance
payment of the proper taxes much needed by the government, monopolies, the LTFRB acted within its prerogatives in
as well as to create a computerized data bank without any cost promoting public interest and protecting the riding public. After
to the government which is necessary for transport planning. all, the consortia are open to all insurance companies, including
The Board will only accept, as proof of compliance of this petitioner. There is no discrimination against any legitimate
program, insurance policies/certificates of cover duly approved insurer. On the whole, the public is given protection without
by the Insurance Commission specifically for this project, and unfair competition or undue restraint of trade. As the Court of
issued by any of the two groups as authorized by the Board. Appeals pointed out, the two consortia are not engaged in the
More so, any insurance company found to have violated any of insurance business; they merely serve as service arms of their
respective members. provisions of R.A No. 8180 and E.O. No. 372.

At bottom, the subject Memorandum Circulars were issued for


the stated purpose of promoting public interest; and of
In G.R. No. 124360, petitioner Francisco S. Tatad seeks the
protecting the riding public and PUV operators from being
annulment of section 5(b) of R.A. No. 8180. Section 5(b)
defrauded by fake, undervalued or misrepresented insurance
provides:
policies.

b) Any law to the contrary notwithstanding and starting


with the effectivity of this Act, tariff duty shall be imposed and
G.R. No. 124360 November 5, 1997 collected on imported crude oil at the rate of three percent
(3%) and imported refined petroleum products at the rate of
seven percent (7%), except fuel oil and LPG, the rate for which
FRANCISCO S. TATAD, petitioner, shall be the same as that for imported crude oil: Provided, That
beginning on January 1, 2004 the tariff rate on imported crude
vs. oil and refined petroleum products shall be the same: Provided,
further, That this provision may be amended only by an Act of
THE SECRETARY OF THE DEPARTMENT OF ENERGY AND THE
Congress.
SECRETARY OF THE DEPARTMENT OF FINANCE, respondents.

In G.R. No. 127867, petitioners Edcel C. Lagman, Joker P.


The petitions at bar challenge the constitutionality of Republic
Arroyo, Enrique Garcia, Wigberto Tanada, Flag Human Rights
Act No. 8180 entitled "An Act Deregulating the Downstream Oil
Foundation, Inc., Freedom from Debt Coalition (FDC) and
Industry and For Other Purposes".
Sanlakas contest the constitutionality of section 15 of R.A. No.
8180 and E.O. No. 392. Section 15 provides:

Prior to 1971, there was no government agency regulating the


oil industry other than those dealing with ordinary
Sec. 15. Implementation of Full Deregulation. — Pursuant to
commodities. Oil companies were free to enter and exit the
Section 5(e) of Republic Act No. 7638, the DOE shall, upon
market without any government interference. All the oil
approval of the President, implement the full deregulation of
refineries and marketing companies were owned by foreigners
the downstream oil industry not later than March 1997. As far
whose economic interests did not always coincide with the
as practicable, the DOE shall time the full deregulation when
interest of the Filipino.
the prices of crude oil and petroleum products in the world
market are declining and when the exchange rate of the peso in
relation to the US dollar is stable. Upon the implementation of
In March 1996, Congress took the audacious step of the full deregulation as provided herein, the transition phase is
deregulating the downstream oil industry. It enacted R.A. No. deemed terminated and the following laws are deemed
8180, entitled the "Downstream Oil Industry Deregulation Act repealed:
of 1996." Under the deregulated environment, "any person or
entity may import or purchase any quantity of crude oil and
petroleum products from a foreign or domestic source, lease or
In assailing section 15 of R.A. No. 8180 and E.O. No. 392,
own and operate refineries and other downstream oil facilities
petitioners offer the following submissions:
and market such crude oil or use the same for his own
requirement," subject only to monitoring by the Department of

Energy. First, section 15 of R.A. No. 8180 constitutes an undue


delegation of legislative power

On February 8, 1997, the President implemented the full


deregulation of the Downstream Oil Industry through E.O. No. Second, petitioners aver that E.O. No. 392 implementing the
372. full deregulation of the downstream oil industry is arbitrary and
unreasonable because it was enacted due to the alleged
depletion of the OPSF fund — a condition not found in R.A. No.
The petitions at bar assail the constitutionality of various 8180.
Third, section 15 of R.A. No. 8180 and E.O. No. 392 allow the (3) Section 9 (b) which states — "To ensure fair competition
formation of a de facto cartel among the three existing oil and prevent cartels and monopolies in the downstream oil
companies — Petron, Caltex and Shell — in violation of the industry, the following acts shall be prohibited:
constitutional prohibition against monopolies, combinations in
restraint of trade and unfair competition.
(b) Predatory pricing which means selling or offering to sell any
product at a price unreasonably below the industry average
Respondents, on the other hand, fervently defend the cost so as to attract customers to the detriment of competitors.
constitutionality of R.A. No. 8180 and E.O. No. 392. In addition,
respondents contend that the issues raised by the petitions are
not justiciable as they pertain to the wisdom of the law. On the other hand, section 19 of Article XII of the Constitution
Respondents further aver that petitioners have no locus standi allegedly violated by the aforestated provisions of R.A. No.
8180 mandates: "The State shall regulate or prohibit
monopolies when the public interest so requires. No
The procedural issues are: (1) whether or not the petitions raise combinations in restraint of trade or unfair competition shall be
a justiciable controversy, and (2) whether or not the petitioners allowed."
have the standing to assail the validity of the subject law and
executive order. The substantive issues are: (1) whether or not
section 5 (b) violates the one title — one subject requirement A monopoly is a privilege or peculiar advantage vested in one
of the Constitution; (2) whether or not the same section or more persons or companies, consisting in the exclusive right
violates the equal protection clause of the Constitution; (3) or power to carry on a particular business or trade,
whether or not section 15 violates the constitutional manufacture a particular article, or control the sale or the
prohibition on undue delegation of power; (4) whether or not whole supply of a particular commodity. It is a form of market
E.O. No. 392 is arbitrary and unreasonable; and (5) whether or structure in which one or only a few firms dominate the total
not R.A. No. 8180 violates the constitutional prohibition against sales of a product or service.28 On the other hand, a
monopolies, combinations in restraint of trade and unfair combination in restraint of trade is an agreement or
competition. understanding between two or more persons, in the form of a
contract, trust, pool, holding company, or other form of
association, for the purpose of unduly restricting competition,
We now come to grips with the contention that some monopolizing trade and commerce in a certain commodity,
provisions of R.A. No. 8180 violate section 19 of Article XII of controlling its, production, distribution and price, or otherwise
the 1987 Constitution. These provisions are: interfering with freedom of trade without statutory
authority.29 Combination in restraint of trade refers to the
means while monopoly refers to the end.30
(1) Section 5 (b) which states — "Any law to the contrary
notwithstanding and starting with the effectivity of this Act,
tariff duty shall be imposed and collected on imported crude oil Article 186 of the Revised Penal Code and Article 28 of the New
at the rate of three percent (3%) and imported refined Civil Code breathe life to this constitutional policy. Article 186
petroleum products at the rate of seven percent (7%) except of the Revised Penal Code penalizes monopolization and
fuel oil and LPG, the rate for which shall be the same as that for creation of combinations in restraint of
imported crude oil. Provided, that beginning on January 1, 2004
trade, 31 while Article 28 of the New Civil Code makes any
the tariff rate on imported crude oil and refined petroleum
person who shall engage in unfair competition liable for
products shall be the same. Provided, further, that this
damages.32
provision may be amended only by an Act of Congress."

Respondents aver that sections 5(b), 6 and 9(b) implement the


(2) Section 6 which states — "To ensure the security and
policies and objectives of R.A. No. 8180. They explain that the
continuity of petroleum crude and products supply, the DOE
4% tariff differential is designed to encourage new entrants to
shall require the refiners and importers to maintain a minimum
invest in refineries. They stress that the inventory requirement
inventory equivalent to ten percent (10%) of their respective
is meant to guaranty continuous domestic supply of petroleum
annual sales volume or forty (40) days of supply, whichever is
and to discourage fly-by-night operators. They also submit that
lower," and
the prohibition against predatory pricing is intended to protect Caltex by building refineries of their own will have to spend
prospective entrants. Respondents manifested to the Court billions of pesos. Those who will not build refineries but
that new players have entered the Philippines after compete with them will suffer the huge disadvantage of
deregulation and have now captured 3% — 5% of the oil increasing their product cost by 4%. They will be competing on
market. an uneven field.

The validity of the assailed provisions of R.A. No. 8180 has to be Finally, we come to the provision on predatory pricing which is
decided in light of the letter and spirit of our Constitution, defined as ". . . selling or offering to sell any product at a price
especially section 19, Article XII. Beyond doubt, the unreasonably below the industry average cost so as to attract
Constitution committed us to the free enterprise system but it customers to the detriment of competitors." Respondents
is a system impressed with its own distinctness. Thus, while the contend that this provision works against Petron, Shell and
Constitution embraced free enterprise as an economic creed, it Caltex and protects new entrants. The ban on predatory pricing
did not prohibit per se the operation of monopolies which can, cannot be analyzed in isolation. Its validity is interlocked with
however, be regulated in the public interest.33 Thus too, our the barriers imposed by R.A. No. 8180 on the entry of new
free enterprise system is not based on a market of pure and players. The inquiry should be to determine whether predatory
unadulterated competition where the State pursues a strict pricing on the part of the dominant oil companies is
hands-off policy and follows the let-the-devil devour the encouraged by the provisions in the law blocking the entry of
hindmost rule. Combinations in restraint of trade and unfair new players. Text-writer
competitions are absolutely proscribed and the proscription is
Hovenkamp,36 gives the authoritative answer and we quote:
directed both against the State as well as the private sector.34
This distinct free enterprise system is dictated by the need to
achieve the goals of our national economy as defined by
section 1, Article XII of the Constitution which are: more The rationale for predatory pricing is the sustaining of losses
equitable distribution of opportunities, income and wealth; a today that will give a firm monopoly profits in the future. The
sustained increase in the amount of goods and services monopoly profits will never materialize, however, if the market
produced by the nation for the benefit of the people; and an is flooded with new entrants as soon as the successful predator
expanding productivity as the key to raising the quality of life attempts to raise its price. Predatory pricing will be profitable
for all, especially the underprivileged. It also calls for the State only if the market contains significant barriers to new entry.
to protect Filipino enterprises against unfair competition and
trade practices.
We come to the final point. We now resolve the total effect of
the untimely deregulation, the imposition of 4% tariff
Section 19, Article XII of our Constitution is anti-trust in history differential on imported crude oil and refined petroleum
and in spirit. It espouses competition. The desirability of products, the requirement of inventory and the prohibition on
competition is the reason for the prohibition against restraint predatory pricing on the constitutionality of R.A. No. 8180. The
of trade, the reason for the interdiction of unfair competition, question is whether these offending provisions can be
and the reason for regulation of unmitigated monopolies. individually struck down without invalidating the entire R.A. No.
Competition is thus the underlying principle of section 19, 8180. The ruling case law is well stated by author Agpalo,37
Article XII of our Constitution which cannot be violated by R.A. viz.:
No. 8180.

The general rule is that where part of a statute is void as


In the cases at bar, it cannot be denied that our downstream oil repugnant to the Constitution, while another part is valid, the
industry is operated and controlled by an oligopoly, a foreign valid portion, if separable from the invalid, may stand and be
oligopoly at that. Petron, Shell and Caltex stand as the only enforced. The presence of a separability clause in a statute
major league players in the oil market. All other players belong creates the presumption that the legislature intended
to the lilliputian league. As the dominant players, Petron, Shell separability, rather than complete nullity of the statute. To
and Caltex boast of existing refineries of various capacities. The justify this result, the valid portion must be so far independent
tariff differential of 4% therefore works to their immense of the invalid portion that it is fair to presume that the
benefit. Yet, this is only one edge of the tariff differential. The legislature would have enacted it by itself if it had supposed
other edge cuts and cuts deep in the heart of their competitors. that it could not constitutionally enact the other. Enough must
It erects a high barrier to the entry of new players. New players remain to make a complete, intelligible and valid statute, which
that intend to equalize the market power of Petron, Shell and carries out the legislative intent. . . .
The exception to the general rule is that when the parts of a In the House of Representatives, the moves to rehabilitate R.A.
statute are so mutually dependent and connected, as No. 8180 are equally feverish. Representative Leopoldo E. San
conditions, considerations, inducements, or compensations for Buenaventura has filed H.B. No. 9826 removing the tariff
each other, as to warrant a belief that the legislature intended differential for imported crude oil and imported refined
them as a whole, the nullity of one part will vitiate the rest. In petroleum products.
making the parts of the statute dependent, conditional, or
connected with one another, the legislature intended the
statute to be carried out as a whole and would not have Representative Marcial C. Punzalan, Jr., filed H.B. No. 9981 to
enacted it if one part is void, in which case if some parts are prevent collusion among the present oil companies by
unconstitutional, all the other provisions thus dependent, strengthening the oversight function of the government
conditional, or connected must fall with them.

Representative Sergio A.F . Apostol filed H.B. No. 10039 to


R.A. No. 8180 contains a separability clause. Section 23 remedy an omission in R.A. No. 8180 where there is no agency
provides that "if for any reason, any section or provision of this in government that determines what is "reasonable" increase
Act is declared unconstitutional or invalid, such parts not in the prices of oil products.
affected thereby shall remain in full force and effect." This
separability clause notwithstanding, we hold that the offending
provisions of R.A. No. 8180 so permeate its essence that the
Representative Dente O. Tinga, one of the principal sponsors of
entire law has to be struck down. The provisions on tariff
R.A. No. 8180, filed H.B. No. 10057 to strengthen its anti-trust
differential, inventory and predatory pricing are among the
provisions.
principal props of R.A. No. 8180. Congress could not have
deregulated the downstream oil industry without these
provisions. Unfortunately, contrary to their intent, these
provisions on tariff differential, inventory and predatory pricing Various resolutions have also been filed in the Senate calling for
inhibit fair competition, encourage monopolistic power and an immediate and comprehensive review of R.A. No. 8180 to
interfere with the free interaction of market forces. R.A. No. prevent the downpour of its ill effects on the people.
8180 needs provisions to vouchsafe free and fair competition.
The need for these vouchsafing provisions cannot be
overstated. Before deregulation, PETRON, SHELL and CALTEX Senator Alberto G. Romulo filed S. Res. No. 769 entitled
had no real competitors but did not have a free run of the resolution "Directing the Committees on Energy and Public
market because government controls both the pricing and non- Services In Aid Of Legislation To Assess The Immediate Medium
pricing aspects of the oil industry. After deregulation, PETRON, And Long Term Impact of Oil Deregulation On Oil Prices And
SHELL and CALTEX remain unthreatened by real competition The Economy."
yet are no longer subject to control by government with
respect to their pricing and non-pricing decisions. The
aftermath of R.A. No. 8180 is a deregulated market where Parallel resolutions have been filed in the House of
competition can be corrupted and where market forces can be Representatives..
manipulated by oligopolies.

With this Decision, some circles will chide the Court for
The fact that the three (3) oil companies' petroleum products interfering with an economic decision of Congress. Such
are uniformly priced suggests collusion, amounting to criticism is charmless for the Court is annulling R.A. No. 8180
cartelization, among Caltex Philippines, Inc., Petron not because it disagrees with deregulation as an economic
Corporation and Pilipinas Shell Petroleum Corporation to fix the policy but because as cobbled by Congress in its present form,
prices of petroleum products in violation of paragraph (a), the law violates the Constitution. The right call therefor should
Section 9 of R.A. No. 8180. be for Congress to write a new oil deregulation law that
conforms with the Constitution and not for this Court to shirk
its duty of striking down a law that offends the Constitution.
Senator Francisco S. Tatad also filed S.B. No. 2307 providing for Striking down R.A. No. 8180 may cost losses in quantifiable
a uniform tariff rate on imported crude oil and refined terms to the oil oligopolists. But the loss in tolerating the
petroleum products. tampering of our Constitution is not quantifiable in pesos and
centavos. More worthy of protection than the supra-normal PLDT filed an Opposition with a Motion to Dismiss, based
profits of private corporations is the sanctity of the multiple grounds among others are the following:
fundamental principles of the Constitution. Indeed when
confronted by a law violating the Constitution, the Court has no 1) ETCI is not capacitated or qualified under its
option but to strike it down dead. Lest it is missed, the legislative franchise to operate a systemwide
Constitution is a covenant that grants and guarantees both the telephone or network of telephone service
political and economic rights of the people. The Constitution 2) the provisional authority, if granted, will result in
needless, uneconomical and harmful duplication
mandates this Court to be the guardian not only of the people's
political rights but their economic rights as well. The protection NTC overruled PLDT's Opposition and declared that Rep.
of the economic rights of the poor and the powerless is of Act No. 2090 (1958) should be liberally construed as to
greater importance to them for they are concerned more with include among the services under said franchise the
the exoterics of living and less with the esoterics of liberty. operation of a cellular mobile telephone service
Hence, for as long as the Constitution reigns supreme so long
On 12 December 1988, NTC granted ETCI provisional
will this Court be vigilant in upholding the economic rights of
authority to install, operate and maintain a cellular mobile
our people especially from the onslaught of the powerful. Our
telephone system initially in Metro Manila subject to the
defense of the people's economic rights may appear heartless terms and conditions set forth. One of the conditions was
because it cannot be half-hearted. that ETCI and PLDT "shall enter into an interconnection
agreement for the provision of adequate interconnection
facilities between applicant's cellular mobile telephone
IN VIEW WHEREOF, the petitions are granted. R.A. No. 8180 is switch and the public switched telephone network and
declared unconstitutional and E.O. No. 372 void. shall jointly submit such interconnection agreement to the
Commission for approval."

In a "Motion to Set Aside the Order", PLDT alleged


SO ORDERED essentially that the interconnection ordered was in
violation of due process and that the grant of provisional
authority was jurisdictionally and procedurally infirm. NTC
denied reconsideration and set the date for continuation of
the hearings on the main proceedings.

PLDT urges us now to annul the NTC Orders of 12


December 1988 and 8 May 1989 and to order ETCI to
PHILIPPINE LONG DISTANCE TELEPHONE CO.
desist from, suspend, and/or discontinue any and all acts
[PLDT], petitioner,
vs. intended for its implementation.
THE NATIONAL TELECOMMUNICATIONS On 27 February 1990, we issued a Temporary Restraining
COMMISSION and CELLCOM, INC., (EXPRESS Order enjoining NTC to "Cease and Desist its on-going
TELECOMMUNICATIONS CO., INC.
proceedings and ETCI and any and all acts which will
[ETCI]), respondents.
amount to the implementation/execution of its provisional
authority."
G.R. No. 88404 October 18, 1990
MELENCIO-HERRERA, J.: The case was set for oral argument on 21 August 1990
with the parties directed to address, but not limited to, the
FACTS: On 22 June 1958, Rep. Act No. 2090, was following issues: (1) the status and coverage of Rep. Act
enacted, otherwise known as "An Act Granting Felix No. 2090 as a franchise; (2) the transfer of shares of stock
Alberto and Company, Incorporated, a Franchise to of a corporation holding a CPCN; and (3) the principle and
Establish Radio Stations for Domestic and Transoceanic procedure of interconnection.
Telecommunications." Felix Alberto & Co., Inc. (FACI) was
the original corporate name, which was changed to ETCI PLDT vehemently opposes interconnection with its own
(CELLCOM was the name to be adopted before the SEC public switched telephone network. While PLDT welcomes
but was abandoned) interconnections, only parties who can establish that they
have valid and subsisting legislative franchises are entitled
On 13 May 1987, alleging urgent public need, ETCI filed to apply for a CPCN or provisional authority; that "the
an application with public respondent NTC for the NTC is not empowered to compel such a private raid
issuance of a Certificate of Public Convenience and on PLDT's legitimate income arising out of its gigantic
Necessity (CPCN) to construct, install, establish, operate
investment;" that "it is not public interest, but purely a
and maintain a Cellular Mobile Telephone System and an
Alpha Numeric Paging System in Metro Manila and in the private and selfish interest which will be served by an
Southern Luzon regions. interconnection under ETCI's terms;" and that "to compel
PLDT to interconnect merely to give viability to a telecommunications services in all areas of the
prospective competitor, which cannot stand on its own Philippines, ... to maximize the use of telecommunications
feet, cannot be justified in the name of a non-existent facilities available, ... in recognition of the vital role of
public need" communications in nation building ... and to ensure that all
users of the public telecommunications service have
access to all other users of the service wherever they may
be within the Philippines at an acceptable standard of
ISSUE: Whether or not the NTC is empowered to compel service and at reasonable cost" (DOTC Circular No. 90-
PLDT, a private company, to enter into an interconnection 248). Undoubtedly, the encompassing objective is the
agreement with ETCI. common good. The NTC, as the regulatory agency of the
State, merely exercised its delegated authority to regulate
the use of telecommunications networks when it decreed
interconnection.
RULING: (1) We find no grave abuse of discretion on the
part of NTC.
The importance and emphasis given to interconnection
Jurisdiction dates back to Ministry Circular No. 82-81, dated 6
December 1982, providing:
The NTC is the regulatory agency of the national
government with jurisdiction over all telecommunications Sec. 1. That the government encourages
entities. It is legally clothed with authority and given ample the provision and operation of public
discretion to grant a provisional permit or authority. mobile telephone service within local sub-
base stations, particularly, in the highly
What the NTC granted was such a provisional authority, commercialized areas;
with a definite expiry period of eighteen (18) months
unless sooner renewed, and which may be revoked, Sec. 5. That, in the event the authority to
amended or revised by the NTC, and was issued after due operate said service be granted to other
hearing, reception of evidence and evaluation thereof, with applicants, other than the franchise
the hearings attended by various oppositors, including holder, the franchise operator shall be
PLDT. under obligation to enter into an
agreement with the domestic telephone
The final outcome of the application rests within the network, under an interconnection
exclusive prerogative of the NTC. agreement;
(2) PLDT cannot justifiably refuse to interconnect.
Department of Transportation and Communication
(DOTC) Circular No. 87-188, issued in 1987, also
Rep. Act No. 6849, or the Municipal Telephone Act of decrees:
1989, approved on 8 February 1990, mandates
interconnection providing as it does that "all domestic
telecommunications carriers or utilities ... shall be 12. All public communications carriers
interconnected to the public switch telephone network." shall interconnect their facilities pursuant
Such regulation of the use and ownership of to comparatively efficient interconnection
telecommunications systems is in the exercise of the (CEI) as defined by the NTC in the
plenary police power of the State for the promotion of the interest of economic efficiency
general welfare. The 1987 Constitution recognizes the
existence of that power when it provides. The NTC order to interconnect allows the parties
themselves to discuss and agree upon the specific terms
SEC. 6. The use of property bears a and conditions of the interconnection agreement instead
social function, and all economic agents of the NTC itself laying down the standards of
shall contribute to the common good. interconnection which it can very well impose. Thus it is
Individuals and private groups, including that PLDT cannot justifiably claim denial of clue process. It
corporations, cooperatives, and similar has been heard. It will continue to be heard in the main
collective organizations, shall have the proceedings. It will surely heard in the negotiations
right to own, establish, and operate concerning the interconnection agreement.
economic enterprises, subject to the duty
of the State to promote distributive justice As disclosed during the hearing, the interconnection
and to intervene when the common good sought by ETCI is by no means a "parasitic dependence"
so demands (Article XII). on PLDT. The ETCI system can operate on its own even
without interconnection, but it will be limited to its own
The interconnection which has been required of PLDT is a subscribers. What interconnection seeks to accomplish is
form of "intervention" with property rights dictated by "the to enable the system to reach out to the greatest number
objective of government to promote the rapid expansion of of people possible in line with governmental policies laid
down. Cellular phones can access PLDT units and vice
versa in as wide an area as attainable. With the broader
reach, public interest and convenience will be better
served.

RATIONALE: Ultimate Considerations

The decisive consideration are public need, public


interest, and the common good. Those were the
overriding factors which motivated NTC in granting
provisional authority to ETCI. Article II, Section 24 of the
1987 Constitution, recognizes the vital role of
communication and information in nation building. It
is likewise a State policy to provide the environment
for the emergence of communications structures
suitable to the balanced flow of information into, out
of, and across the country (Article XVI, Section
10, Ibid.). A modern and dependable communications
network rendering efficient and reasonably priced services
is also indispensable for accelerated economic recovery
and development. To these public and national interests,
public utility companies must bow and yield.

Additionally, the State is empowered to decide


whether public interest demands that monopolies be
regulated or prohibited (1987 Constitution. Article XII,
Section 19).

DISPOSITIVE PART: WHEREFORE, finding no grave


abuse of discretion, tantamount to lack of or excess of
jurisdiction, on the part of the National
Telecommunications Commission in issuing its challenged
Orders of 12 December 1988 and 8 May 1989 in NTC
Case No. 87-39, this Petition is DISMISSED for lack of
merit. The Temporary Restraining Order heretofore issued
is LIFTED. The bond issued as a condition for the
issuance of said restraining Order is declared forfeited in
favor of private respondent Express Telecommunications
Co., Inc. Costs against petitioner.

You might also like